How can a planet have a deadly eclipse-like “spotlight”?












22














A solar eclipse occurs when the moon passes between the Earth and the sun. The result is a giant shadow that sweeps across the Earth's surface.



This world has the opposite phenomenon. Instead of a giant shadow, this world has a giant deadly "spotlight" sweep across its surface.



Must achieve these effects:




  1. Must produce enough thermal power to kill humans (who are not native) and some (or all) non-native animals efficiently enough that full exposure to the light is (near?) certain doom.

  2. Should have easily-observed warning signs allowing vulnerable creatures roughly one minute to find shelter. None of the lifeforms on the planet have developed its meteorology well enough to predict them except by visual/thermal observation.

  3. Must be natural/meteorological/astronomical in nature. (Nothing like a giant orbiting laser).

  4. Most or all plants, fungi, etc (non-animals) should be able to survive.


These would be a plus:




  1. The starlight should normally be white, but the spotlight should be red (best case), blue (next best) or orange (third best). Otherwise, just brighter.

  2. Should occur a few times per week in at least one area on the planet.

  3. Should not be precisely periodic. If two occurrences are 36 hours apart, the next one might be 34 hours after, or 50 hours after. There may be a complex pattern, but it shouldn't be plausible to figure out over the course of a couple of weeks.


A "spotlight" that covers part of the planet is preferred, but if being larger than the planet is easier, that may be acceptable.



Here are a few ideas I was toying with, but I'm not sure how realistic they are:




  1. There are multiple small (or distant) stars in the system, each white in color, and multiple red moons surrounding the planet. The stars are clustered around the center of the system so there is still a day/night cycle (is this possible?) with a roughly Earthlike luminosity. The red moons reflect light on the surface more or less constantly but the intensity is relatively small compared to the normal light from the stars... Until enough moons reflect light from enough stars to the same spot on the planet's surface! I don't know if they can really reflect enough light for this to work though.

  2. Because of chaotic atmospheric conditions, atmospheric refraction causes the (single) star's rays to focus on a small area. I don't know if the color objective can be accomplished this way though.

  3. There is very thick cloud cover, but sometimes a hole opens that allows the full light of the star through onto part of the surface. Also not sure if the color objective can be accomplished.










share|improve this question




















  • 1




    Natural is strongly preferred.
    – Devsman
    yesterday






  • 5




    I'm thinking something like a Dyson sphere around a very bright star, with one or more holes in it that periodically sweep over the planet. Normally the star appears to be very large and dim (as what you can see is actually the glowing outer surface of the Dyson sphere), but occasionally a hole rotates into position to scorch a path across the planet. It's technically artificial but could have been abandonened hundreds of millions of years earlier.
    – Gryphon
    yesterday








  • 2




    Conservation of etendue. Can't have a moon that is brighter than its parent star using just ref*ction. The bigger issue is producing the spotlight effect though.
    – John Dvorak
    yesterday






  • 12




    Scratch that. Devising anything that kills animals reliably while leaving plants untouched is pretty much impossible. For one thing, you'd have to explain why the animals don't just fancy a coat of lichen from day zero of their evolution.
    – John Dvorak
    yesterday








  • 1




    @JohnDvorak Point taken. I'll update so it's not necessary for native animals to be killed.
    – Devsman
    yesterday
















22














A solar eclipse occurs when the moon passes between the Earth and the sun. The result is a giant shadow that sweeps across the Earth's surface.



This world has the opposite phenomenon. Instead of a giant shadow, this world has a giant deadly "spotlight" sweep across its surface.



Must achieve these effects:




  1. Must produce enough thermal power to kill humans (who are not native) and some (or all) non-native animals efficiently enough that full exposure to the light is (near?) certain doom.

  2. Should have easily-observed warning signs allowing vulnerable creatures roughly one minute to find shelter. None of the lifeforms on the planet have developed its meteorology well enough to predict them except by visual/thermal observation.

  3. Must be natural/meteorological/astronomical in nature. (Nothing like a giant orbiting laser).

  4. Most or all plants, fungi, etc (non-animals) should be able to survive.


These would be a plus:




  1. The starlight should normally be white, but the spotlight should be red (best case), blue (next best) or orange (third best). Otherwise, just brighter.

  2. Should occur a few times per week in at least one area on the planet.

  3. Should not be precisely periodic. If two occurrences are 36 hours apart, the next one might be 34 hours after, or 50 hours after. There may be a complex pattern, but it shouldn't be plausible to figure out over the course of a couple of weeks.


A "spotlight" that covers part of the planet is preferred, but if being larger than the planet is easier, that may be acceptable.



Here are a few ideas I was toying with, but I'm not sure how realistic they are:




  1. There are multiple small (or distant) stars in the system, each white in color, and multiple red moons surrounding the planet. The stars are clustered around the center of the system so there is still a day/night cycle (is this possible?) with a roughly Earthlike luminosity. The red moons reflect light on the surface more or less constantly but the intensity is relatively small compared to the normal light from the stars... Until enough moons reflect light from enough stars to the same spot on the planet's surface! I don't know if they can really reflect enough light for this to work though.

  2. Because of chaotic atmospheric conditions, atmospheric refraction causes the (single) star's rays to focus on a small area. I don't know if the color objective can be accomplished this way though.

  3. There is very thick cloud cover, but sometimes a hole opens that allows the full light of the star through onto part of the surface. Also not sure if the color objective can be accomplished.










share|improve this question




















  • 1




    Natural is strongly preferred.
    – Devsman
    yesterday






  • 5




    I'm thinking something like a Dyson sphere around a very bright star, with one or more holes in it that periodically sweep over the planet. Normally the star appears to be very large and dim (as what you can see is actually the glowing outer surface of the Dyson sphere), but occasionally a hole rotates into position to scorch a path across the planet. It's technically artificial but could have been abandonened hundreds of millions of years earlier.
    – Gryphon
    yesterday








  • 2




    Conservation of etendue. Can't have a moon that is brighter than its parent star using just ref*ction. The bigger issue is producing the spotlight effect though.
    – John Dvorak
    yesterday






  • 12




    Scratch that. Devising anything that kills animals reliably while leaving plants untouched is pretty much impossible. For one thing, you'd have to explain why the animals don't just fancy a coat of lichen from day zero of their evolution.
    – John Dvorak
    yesterday








  • 1




    @JohnDvorak Point taken. I'll update so it's not necessary for native animals to be killed.
    – Devsman
    yesterday














22












22








22


1





A solar eclipse occurs when the moon passes between the Earth and the sun. The result is a giant shadow that sweeps across the Earth's surface.



This world has the opposite phenomenon. Instead of a giant shadow, this world has a giant deadly "spotlight" sweep across its surface.



Must achieve these effects:




  1. Must produce enough thermal power to kill humans (who are not native) and some (or all) non-native animals efficiently enough that full exposure to the light is (near?) certain doom.

  2. Should have easily-observed warning signs allowing vulnerable creatures roughly one minute to find shelter. None of the lifeforms on the planet have developed its meteorology well enough to predict them except by visual/thermal observation.

  3. Must be natural/meteorological/astronomical in nature. (Nothing like a giant orbiting laser).

  4. Most or all plants, fungi, etc (non-animals) should be able to survive.


These would be a plus:




  1. The starlight should normally be white, but the spotlight should be red (best case), blue (next best) or orange (third best). Otherwise, just brighter.

  2. Should occur a few times per week in at least one area on the planet.

  3. Should not be precisely periodic. If two occurrences are 36 hours apart, the next one might be 34 hours after, or 50 hours after. There may be a complex pattern, but it shouldn't be plausible to figure out over the course of a couple of weeks.


A "spotlight" that covers part of the planet is preferred, but if being larger than the planet is easier, that may be acceptable.



Here are a few ideas I was toying with, but I'm not sure how realistic they are:




  1. There are multiple small (or distant) stars in the system, each white in color, and multiple red moons surrounding the planet. The stars are clustered around the center of the system so there is still a day/night cycle (is this possible?) with a roughly Earthlike luminosity. The red moons reflect light on the surface more or less constantly but the intensity is relatively small compared to the normal light from the stars... Until enough moons reflect light from enough stars to the same spot on the planet's surface! I don't know if they can really reflect enough light for this to work though.

  2. Because of chaotic atmospheric conditions, atmospheric refraction causes the (single) star's rays to focus on a small area. I don't know if the color objective can be accomplished this way though.

  3. There is very thick cloud cover, but sometimes a hole opens that allows the full light of the star through onto part of the surface. Also not sure if the color objective can be accomplished.










share|improve this question















A solar eclipse occurs when the moon passes between the Earth and the sun. The result is a giant shadow that sweeps across the Earth's surface.



This world has the opposite phenomenon. Instead of a giant shadow, this world has a giant deadly "spotlight" sweep across its surface.



Must achieve these effects:




  1. Must produce enough thermal power to kill humans (who are not native) and some (or all) non-native animals efficiently enough that full exposure to the light is (near?) certain doom.

  2. Should have easily-observed warning signs allowing vulnerable creatures roughly one minute to find shelter. None of the lifeforms on the planet have developed its meteorology well enough to predict them except by visual/thermal observation.

  3. Must be natural/meteorological/astronomical in nature. (Nothing like a giant orbiting laser).

  4. Most or all plants, fungi, etc (non-animals) should be able to survive.


These would be a plus:




  1. The starlight should normally be white, but the spotlight should be red (best case), blue (next best) or orange (third best). Otherwise, just brighter.

  2. Should occur a few times per week in at least one area on the planet.

  3. Should not be precisely periodic. If two occurrences are 36 hours apart, the next one might be 34 hours after, or 50 hours after. There may be a complex pattern, but it shouldn't be plausible to figure out over the course of a couple of weeks.


A "spotlight" that covers part of the planet is preferred, but if being larger than the planet is easier, that may be acceptable.



Here are a few ideas I was toying with, but I'm not sure how realistic they are:




  1. There are multiple small (or distant) stars in the system, each white in color, and multiple red moons surrounding the planet. The stars are clustered around the center of the system so there is still a day/night cycle (is this possible?) with a roughly Earthlike luminosity. The red moons reflect light on the surface more or less constantly but the intensity is relatively small compared to the normal light from the stars... Until enough moons reflect light from enough stars to the same spot on the planet's surface! I don't know if they can really reflect enough light for this to work though.

  2. Because of chaotic atmospheric conditions, atmospheric refraction causes the (single) star's rays to focus on a small area. I don't know if the color objective can be accomplished this way though.

  3. There is very thick cloud cover, but sometimes a hole opens that allows the full light of the star through onto part of the surface. Also not sure if the color objective can be accomplished.







astronomy weather






share|improve this question















share|improve this question













share|improve this question




share|improve this question








edited yesterday







Devsman

















asked yesterday









DevsmanDevsman

2,7651725




2,7651725








  • 1




    Natural is strongly preferred.
    – Devsman
    yesterday






  • 5




    I'm thinking something like a Dyson sphere around a very bright star, with one or more holes in it that periodically sweep over the planet. Normally the star appears to be very large and dim (as what you can see is actually the glowing outer surface of the Dyson sphere), but occasionally a hole rotates into position to scorch a path across the planet. It's technically artificial but could have been abandonened hundreds of millions of years earlier.
    – Gryphon
    yesterday








  • 2




    Conservation of etendue. Can't have a moon that is brighter than its parent star using just ref*ction. The bigger issue is producing the spotlight effect though.
    – John Dvorak
    yesterday






  • 12




    Scratch that. Devising anything that kills animals reliably while leaving plants untouched is pretty much impossible. For one thing, you'd have to explain why the animals don't just fancy a coat of lichen from day zero of their evolution.
    – John Dvorak
    yesterday








  • 1




    @JohnDvorak Point taken. I'll update so it's not necessary for native animals to be killed.
    – Devsman
    yesterday














  • 1




    Natural is strongly preferred.
    – Devsman
    yesterday






  • 5




    I'm thinking something like a Dyson sphere around a very bright star, with one or more holes in it that periodically sweep over the planet. Normally the star appears to be very large and dim (as what you can see is actually the glowing outer surface of the Dyson sphere), but occasionally a hole rotates into position to scorch a path across the planet. It's technically artificial but could have been abandonened hundreds of millions of years earlier.
    – Gryphon
    yesterday








  • 2




    Conservation of etendue. Can't have a moon that is brighter than its parent star using just ref*ction. The bigger issue is producing the spotlight effect though.
    – John Dvorak
    yesterday






  • 12




    Scratch that. Devising anything that kills animals reliably while leaving plants untouched is pretty much impossible. For one thing, you'd have to explain why the animals don't just fancy a coat of lichen from day zero of their evolution.
    – John Dvorak
    yesterday








  • 1




    @JohnDvorak Point taken. I'll update so it's not necessary for native animals to be killed.
    – Devsman
    yesterday








1




1




Natural is strongly preferred.
– Devsman
yesterday




Natural is strongly preferred.
– Devsman
yesterday




5




5




I'm thinking something like a Dyson sphere around a very bright star, with one or more holes in it that periodically sweep over the planet. Normally the star appears to be very large and dim (as what you can see is actually the glowing outer surface of the Dyson sphere), but occasionally a hole rotates into position to scorch a path across the planet. It's technically artificial but could have been abandonened hundreds of millions of years earlier.
– Gryphon
yesterday






I'm thinking something like a Dyson sphere around a very bright star, with one or more holes in it that periodically sweep over the planet. Normally the star appears to be very large and dim (as what you can see is actually the glowing outer surface of the Dyson sphere), but occasionally a hole rotates into position to scorch a path across the planet. It's technically artificial but could have been abandonened hundreds of millions of years earlier.
– Gryphon
yesterday






2




2




Conservation of etendue. Can't have a moon that is brighter than its parent star using just ref*ction. The bigger issue is producing the spotlight effect though.
– John Dvorak
yesterday




Conservation of etendue. Can't have a moon that is brighter than its parent star using just ref*ction. The bigger issue is producing the spotlight effect though.
– John Dvorak
yesterday




12




12




Scratch that. Devising anything that kills animals reliably while leaving plants untouched is pretty much impossible. For one thing, you'd have to explain why the animals don't just fancy a coat of lichen from day zero of their evolution.
– John Dvorak
yesterday






Scratch that. Devising anything that kills animals reliably while leaving plants untouched is pretty much impossible. For one thing, you'd have to explain why the animals don't just fancy a coat of lichen from day zero of their evolution.
– John Dvorak
yesterday






1




1




@JohnDvorak Point taken. I'll update so it's not necessary for native animals to be killed.
– Devsman
yesterday




@JohnDvorak Point taken. I'll update so it's not necessary for native animals to be killed.
– Devsman
yesterday










12 Answers
12






active

oldest

votes


















42














A wobbling pulsar will do the trick.



Pulsars emit a lot of energy in narrow beams that come from their poles. The slowest ones flash every few seconds; make its tilt wobble so that it is not pointing at the planet all the time. In addition, wobbling causes the pulsar to shoot at different points of the planet's orbit through time. The planet is hit when the pulsar's beams' path just happens to be passing by the planet.



If the pulsar is flashing every few milliseconds (as is normal for them), it will seem like a continuous beam for observers.



Finally, to make the beam small enough that it doesn't cover the entire planet and more, justify it with lensing from nearby nebulas, the planet's atmosphere, and maybe a black hole between the pulsar and the planet.






share|improve this answer



















  • 2




    Gah, I can't believe I didn't think of this. You got my +1.
    – Gryphon
    yesterday






  • 6




    +1, even easier if it CAN cover the whole planet, which I think might work for the story.
    – Devsman
    yesterday






  • 1




    Pulsars are stars that have previously exploded as supernovae: explaining life on a planet after a supernova is either a problem or interesting, depending on how you look at it.
    – Phil Frost
    yesterday






  • 3




    @Renan Meaning the pulsar isn't the center of the system but just making a pass nearby? Yeah, that could work. en.wikipedia.org/wiki/Barnard%27s_Star#/media/…
    – Phil Frost
    yesterday






  • 8




    Clarification (you might already understand this, but it is worth clarifying): pulsars don't "fire". They constantly emit their beam of light as long as the neutron star is accreting matter. The reason why they "blink" from our perspective is because the beam of light is focused (as you mention) and therefore only crosses our path once per rotation - not because it turns on and off. It's always on. The "period" is determined by its rotation rate, and doesn't have to be fast.
    – conman
    yesterday



















21














The remains of an ancient Dyson swarm



Not quite natural, but mostly non-technological. If a prior civilization had constructed a Dyson swarm around the system's star, the light coming from the star might be heavily occluded. Assume the sun is >10X hotter than ours (or the planet is much closer in), and there are enough collector bodies in the swarm to block some 90% of the sun's light at any time. If they are close enough in toward the sun, there will be enough diffraction around each collector that they wouldn't cast visible shadows, and could only be observed by direct observation of the sun, which requires a minimum level of technology to avoid blinding yourself.



The spotlight effect would occur when resonances in the orbital periods of the different bodies in the swarm cause gaps in coverage. The creating civilization could have arranged this purposefully to provide sunlight to further flung planets/stations, or be coincidental. The apparent brightness would grow gradually as more pieces of the swarm leave the "hole" in the field, so the warning sign would just be a rapid but gradual increase in brightness.



The spotlight color would probably be the same as the normal sunlight. However, if the star is very hot, heading toward blue spectrum, the swarm might occlude the blue/UV portion of the spectrum more and let redder light through (imagine if each collector is a giant solar array panel with no backing, e.g. microns of silicon). At the very least they would radiate heat in the infrared. If natives to the planet are used to these conditions, that might be their normal "white" light.






share|improve this answer





















  • Perhaps the gaps in coverage are caused by swarm members that have failed and fell out of orbit (shot down by a solar flare, by resonance with a heavy planet, collision with an in-falling comet...).
    – John Dvorak
    yesterday



















9














Any sort of passive light-focusing (with lenses, mirrors, etc) scheme is unlikely do more than to make slightly warm spots. The fundamental reason has to do with the conservation of etendue, and you can read more about it at Would a Moon made of water pose a threat to Earth during eclipses?



As such, if you want the spotlight to come from a moon, the moon would either need some kind of power source (which starts to sound like "giant lasers") or would need some natural mechanism to eject jets of energy or matter. As far as I know, all kinds of astrophysical jets would require something much more massive than a moon, so this seems like a dead-end.



I think the most feasible explanation is a planet which is ordinarily protected by its atmosphere and/or magnetosphere, but on occasion the weather aligns such that the protection is lost in an area. We to experience this to a small extent on earth: both the sun and earth have magnetic fields that vary over time. One trouble is if the earth's magnetosphere were periodically penetrated by the solar wind, the atmosphere would be stripped away. Though it could take a very long time -- perhaps it is interesting for your story to have a "dying planet".






share|improve this answer





















  • +1. If it enables the scenario, a dying planet is perfectly fine. It doesn't matter what happens to the planet after the story ends. :P
    – Devsman
    yesterday



















4














Not entirely sure about the feasibility of this, but it's an idea I had when I read your question. I wonder if something like this would be possible through Gravitational Lensing. Essentially, this is when black holes (with enormous gravitational pulls) bend light around them, causing telescopic effects. I've linked the Wikipedia page for gravitational lensing as well as an article from Space.com below which you could read up on to give you a better idea of how it all works.



My idea though is that what if, just outside the edge of what can be seen from the planet, there's a system of black holes which pull light in such a manner that it's focussed into a thin beam, which cuts across the galaxy and occasionally burns its way across your planet? This would explain the huge intensity of the light as well as allowing a 'natural' explanation for how it's focussed so tightly.



Having it at such a distance would also mean that the appearance of the beam of light can't be predicted, as the people on the planet don't have the technology to either see that far into space, or understand what they're seeing. Besides, at such a distance that the black holes don't mess with the solar system's structure, the light would take very long to reach the planet. So when the beam lines up, its effects are only seen on the planet later (how much later depends on the distance. Centuries, or even millennia maybe).



As for the colour of the beam, we can assume the source of light is moving away from the black holes and the planet, which would cause Redshift. This would make a white light source look red to the observer. Frequency wise, we could assume that other objects in space (planets, dust clouds etc) often block the light from hitting our planet, but occasionally it slips through the gaps (like when you see the sun for a couple of seconds through a clearing in the clouds before it gets blocked again).



For non-animals to survive, perhaps they've evolved to feed on the huge light intensity and maybe even need it every few days to live? Or (depending on the history of the 'humans' on your planet) maybe everything else is evolved to survive the intense light to an extent, while humans aren't. Perhaps this is similar to how we have to wear clothes - we can't handle Earth's natural climates without external help. Maybe those caught outside of their radiation booths are killed rapidly, while those who stay inside are fine?



There's a lot of ways you can go with this idea and I think the rest is up to you. I've included a few links at the bottom you might be interested in.



Further Reading:



Gravitational lensing:



https://www.space.com/39999-how-gravitational-lenses-work.html



https://en.wikipedia.org/wiki/Gravitational_lens



Red and blue shift:



https://en.wikipedia.org/wiki/Redshift



https://www.space.com/25732-redshift-blueshift.html






share|improve this answer





















  • +1 for redshift
    – Devsman
    yesterday






  • 8




    Sadly, the conservation of etendue makes this system of black hole lenses unworkable. If you're concentrating the light to a smaller output area, then it must also be distributed across a larger angle as it leaves the lens. XKCD's Randall Munroe did an excellent post on this topic: what-if.xkcd.com/145
    – Dubukay
    yesterday






  • 1




    @Dubukay wow, never heard of that conservation law before. Thanks for bringing it to my attention! Love learning stuff on here haha
    – user43712
    yesterday






  • 4




    However, if what we're going for is suspension-of-disbelief, make it a mini black hole in an inner orbit of the same system. Each time that BH passes between your unfortunate planet and the sun, lensing burns a scar right across the planet. If there's enough sunlight to start a fire with a magnifying glass, there's enough to do it through gravitational lensing (note that most of the planet will be nearly dark due to the lensing effect).
    – JBH
    yesterday






  • 5




    Also, keep in mind that the mini black hole could also be one half of a binary star, in which case the orbits are preserved and the effect is the same, if a bit more complicated to calculate.
    – JBH
    yesterday



















2














Ozone holes (https://en.wikipedia.org/wiki/Ozone_depletion) already cause real world health problems.



I think you could combine ozone holes and the loss of other atmospheric protection with the a coinciding local weakening of the magnetosphere. In the presence of a very energetic "sun", the results might be "deadly".



"Deadly" as in it might be immediately painful and certainly cancerous over time. Not deadly in the Vin Diesel kind of way you might be looking for though.






share|improve this answer





















  • A magnetosphere has no effect on electromagnetic radiation, and a breathable atmosphere would protect against solar wind.
    – Christopher James Huff
    yesterday



















2














Let's have some cosmic fun.



Say that your planet orbits inside of an expanding red giant star.



Yep.



Now, say that someone or something decided to construct a shell around your world, perhaps before it had been engulfed by your host star as it evolved from the main sequence. This shell may be layers upon layers of orbital rings.



Now, say that this shell--made of some impossible material, probably--has some degree of translucency to it and the ability to change this translucency. You can have one hemisphere of the shell totally opaque to simulate night, with perhaps little points of translucency to simulate stars, and the other hemisphere translucent to simulate daylight (much less a simulation this point, because actual starlight would be entering).



Finally, let this shell have some circular region of total transparency which sweeps the planet, perhaps moving across the shell faster than the day-night cycle to give the folks on all sides, day or night, some of the action. You can think of the shell as being comprised of a bunch of little windows if you'd like, like pixels on a screen, that can be tinted and whatnot and can be made to filter out the unnecessary or harmful radiation of the star it is orbiting inside. The transparent portion would not filter these things, allowing death to beam down onto everything below. Heck, perhaps it even has little perforations or 'openings of the windows' that allow actual, high-energy particles to enter as well.






share|improve this answer



















  • 1




    "Say that your planet orbits inside of an expanding red giant star." It would quickly deorbit due to friction.
    – Renan
    yesterday






  • 1




    @Renan Not as quickly as you'd think.
    – B.fox
    yesterday










  • @B.fox, but still very quickly on a scale of planets. A very rough approximation, pretending that the classic drag equation is valid at all speeds, is that an Earth-sized planet will come to a complete stop after about 400 years.
    – Mark
    yesterday












  • @Mark I'll have to follow up on that. I'm pretty sure the range was somewhere in the hundreds of thousands of years. The outer atmosphere of a red giant star is pretty diffuse, almost insubstantial.
    – B.fox
    22 hours ago












  • I'm not sure why this solution should depend on the planet being inside the sun. Take our own system, build such a shell around Mercury.
    – Mr Lister
    12 hours ago





















1














Constant lightning.



Consider a Rocheworld.
Can an atmosphere englobe a planetary ring?




Two tidally locked planets just outside the Roche Limit can orbit each
other and share a combined atmosphere. You would be able to fly from
one to the other without ever leaving the atmosphere and objects
placed at the lagrange points would be able to remain there.




These binary planets circle around each other. At one point in their orbit, their atmospheres touch (or you could have a moon graze the atmosphere of its planet). The friction of the two atmospheres against each other produces colossal amounts of electrical charge. When the two are close enough that the atmosphere can provide a path, charge can equilibrate across.



This takes the form of constant tremendous bolts of lightning that follow the path of the point where the two partners are closest to one another.






share|improve this answer





























    1














    I haven't seen this directly addressed, so I'll pose it as an answer:



    TL;DR: Large moon with atmosphere refracts to a "point" on your planet. See below for etendue/thermodynamics, refraction, periodicity, and "warning signs".



    A companion (moon, twin planet, or even planet as primary with your story set on a comfortable moon of a gas giant) large enough to hold a substantial atmosphere can perhaps be tuned to get the result you need.



    Devise an atmosphere for the companion body with a powerful thermal inversion somewhere that reduces some of the spreading due to typical refraction of a density-stratified lens.



    So we effectively have a ring-shaped lens, fairly narrow (edge view of the companion's atmosphere) but of very wide diameter (the companion itself), tuned to refract fairly well to a "point". The source of the light is the sun, and we will not get hotter than that. We do not need perfect point focus, but will gladly accept a central line of foci for various degrees of refraction, which generate -- you guessed it -- different colors of spotlight at different orbital distance of the large body from your planet's surface. Blue when it's close, red when it's far -- if it behaves like a proper lens-shaped lens. This would also result in color change as the effect sweeps from the edge of the home planet (farther) to the center (a bit closer). Warning signs would be similar to normal eclipses (the effect would only be observable from the very height of the eclipse). Finally, a combination of rotational and orbital planes for the three bodies involved can do wonders for making a simple periodic set of processes appear miserably non-periodic, particularly for observers located at different points on the surface of the home body.



    I'll create a graphic. But I think this thing is doable with a lot less machinery than has been proposed so far, and without violating physics to the point of ridicule.






    share|improve this answer



















    • 1




      The only reason this won't get ridiculed for violating physics is that most people don't know about conservation of etendue.
      – Mark
      yesterday










    • Mark, how so? A garden variety magnifying glass does not get as hot as the point focused upon.
      – Haakon Dahl
      7 mins ago



















    0














    The moon discussion referenced by @Phil Frost suggests part of the answer. A moon is too small so the lens-like body or phenomenon has to be big enough to cover all or most of the sky from the point of view of the target planet (which may itself be just a moon in a bigger system).



    The problem is coming up with a celestial lens. If you can solve that, the rest is just a question of placing the target planet and the radiation source at a suitable scale and proximity.



    A lens spotlight redirects light from a large area outside the "spot" so the first warning of the death ray's proximity would be a significant darkening, similar to a solar eclipse. In the distance you might see reflections from dust or clouds within the cone of concentrated light, so you can see if it's coming closer.






    share|improve this answer








    New contributor




    maxwellsdemon is a new contributor to this site. Take care in asking for clarification, commenting, and answering.
    Check out our Code of Conduct.


















    • You would also see the rim of the magnified sun start to appear from one side as the danger area approached, and could tell if it was going to pass you by or go right over you. But focusing light in such a way requires arrangements of matter too specifically contrived for nature to be a believable explanation. Your best bet would seem to be along the lines of neglectful precursors.
      – Christopher James Huff
      yesterday



















    0














    The system could be a binary system with a neutron star or black hole orbiting close to the main star but in an eccentric orbit that lasts a few days. When it draws close to the primary it pulls off huge masses of coronal gasses and causes massive incredibly intense solar flares. If these incidents happen at the same time as the planet is in the wrong part of the sky then you can expect some serious pyrotechnics to hit the day side of the planet for a few hours.



    The black hole/neutron star would most likely have been captured rather than be an original part of the system, explaining the eccentric orbit and any unusual spin needed etc.



    It doesn't take a big stretch to somehow say that the gravity and magnetic fields of the neutron star focuses the ejections into beams somehow. So every X hours you get massive beams of solar energy being fired in random directions. You can then get the variance by saying whether those beams hit your planet or not.






    share|improve this answer





















    • I think this is too deadly. The situation you describe is similar to a classic nova, and the 10,000-fold increase in solar output from one of those will quite handily sterilize a planet.
      – Mark
      12 hours ago



















    -2














    Reverse Solar Eclipse.

    An orbiting Ocean, say a liquid helium moon of the correct size. Meaning, blocking the entire star, or larger. It could create a focusing and penumbra effect based on your need for destruction. Reinforce your physics.






    share|improve this answer










    New contributor




    SiGGER is a new contributor to this site. Take care in asking for clarification, commenting, and answering.
    Check out our Code of Conduct.














    • 2




      The correct size is roughly "fills the entire sky". See conservation of etendue, XKCD What-If #145, and the answers to this question.
      – Mark
      yesterday



















    -3














    A transparent sphere works as a burning glass so a moon of (impossible) clear material should do the trick by concentrating the rays from the sun if it orbited at the right distance.



    Trouble is that absorbtion would eat most of the light if the diameter was more than a kilometer. A thin ice-shell might work, but good luck with explaining the origin (and stability!!) of that ;-)






    share|improve this answer








    New contributor




    Mads Horn is a new contributor to this site. Take care in asking for clarification, commenting, and answering.
    Check out our Code of Conduct.


















    • Welcome to Worldbuilding, Mads Horn! If you have a moment, please take the tour and visit the help center to learn more about the site. You may also find Worldbuilding Meta and The Sandbox useful. Here is a meta post on the culture and style of Worldbuilding.SE, just to help you understand our scope and methods, and how we do things here. Have fun!
      – Gryphon
      yesterday






    • 1




      A thin layer of ice may let enough light through, but it won't have any significant lensing effect, not to mention it wouldn't survive without collapsing spectacularly for more than a few days after being conjured at best, let alone form naturally in the first place.
      – John Dvorak
      yesterday






    • 1




      "If it orbited at the right distance": the focal distance of a ball lens is $f = nD / 4(n - 1)$, with n being the index of refraction of the material and D the diameter of the sphere. For glass, this works out at about 0.8 D, so that orbit must be very close to the surface. Not to mention that the focus lies on the optical axis, so that it wont fall on the surface unless the moon is in conjuction with the Sun. And ball lenses are horrible lenses, they won't focus the light in a nice focal spot.
      – AlexP
      yesterday








    • 6




      Won't work, for reasons more than just absorption. See Would a Moon made of water pose a threat to Earth during eclipses?
      – Phil Frost
      yesterday













    Your Answer





    StackExchange.ifUsing("editor", function () {
    return StackExchange.using("mathjaxEditing", function () {
    StackExchange.MarkdownEditor.creationCallbacks.add(function (editor, postfix) {
    StackExchange.mathjaxEditing.prepareWmdForMathJax(editor, postfix, [["$", "$"], ["\\(","\\)"]]);
    });
    });
    }, "mathjax-editing");

    StackExchange.ready(function() {
    var channelOptions = {
    tags: "".split(" "),
    id: "579"
    };
    initTagRenderer("".split(" "), "".split(" "), channelOptions);

    StackExchange.using("externalEditor", function() {
    // Have to fire editor after snippets, if snippets enabled
    if (StackExchange.settings.snippets.snippetsEnabled) {
    StackExchange.using("snippets", function() {
    createEditor();
    });
    }
    else {
    createEditor();
    }
    });

    function createEditor() {
    StackExchange.prepareEditor({
    heartbeatType: 'answer',
    autoActivateHeartbeat: false,
    convertImagesToLinks: false,
    noModals: true,
    showLowRepImageUploadWarning: true,
    reputationToPostImages: null,
    bindNavPrevention: true,
    postfix: "",
    imageUploader: {
    brandingHtml: "Powered by u003ca class="icon-imgur-white" href="https://imgur.com/"u003eu003c/au003e",
    contentPolicyHtml: "User contributions licensed under u003ca href="https://creativecommons.org/licenses/by-sa/3.0/"u003ecc by-sa 3.0 with attribution requiredu003c/au003e u003ca href="https://stackoverflow.com/legal/content-policy"u003e(content policy)u003c/au003e",
    allowUrls: true
    },
    noCode: true, onDemand: true,
    discardSelector: ".discard-answer"
    ,immediatelyShowMarkdownHelp:true
    });


    }
    });














    draft saved

    draft discarded


















    StackExchange.ready(
    function () {
    StackExchange.openid.initPostLogin('.new-post-login', 'https%3a%2f%2fworldbuilding.stackexchange.com%2fquestions%2f135840%2fhow-can-a-planet-have-a-deadly-eclipse-like-spotlight%23new-answer', 'question_page');
    }
    );

    Post as a guest















    Required, but never shown

























    12 Answers
    12






    active

    oldest

    votes








    12 Answers
    12






    active

    oldest

    votes









    active

    oldest

    votes






    active

    oldest

    votes









    42














    A wobbling pulsar will do the trick.



    Pulsars emit a lot of energy in narrow beams that come from their poles. The slowest ones flash every few seconds; make its tilt wobble so that it is not pointing at the planet all the time. In addition, wobbling causes the pulsar to shoot at different points of the planet's orbit through time. The planet is hit when the pulsar's beams' path just happens to be passing by the planet.



    If the pulsar is flashing every few milliseconds (as is normal for them), it will seem like a continuous beam for observers.



    Finally, to make the beam small enough that it doesn't cover the entire planet and more, justify it with lensing from nearby nebulas, the planet's atmosphere, and maybe a black hole between the pulsar and the planet.






    share|improve this answer



















    • 2




      Gah, I can't believe I didn't think of this. You got my +1.
      – Gryphon
      yesterday






    • 6




      +1, even easier if it CAN cover the whole planet, which I think might work for the story.
      – Devsman
      yesterday






    • 1




      Pulsars are stars that have previously exploded as supernovae: explaining life on a planet after a supernova is either a problem or interesting, depending on how you look at it.
      – Phil Frost
      yesterday






    • 3




      @Renan Meaning the pulsar isn't the center of the system but just making a pass nearby? Yeah, that could work. en.wikipedia.org/wiki/Barnard%27s_Star#/media/…
      – Phil Frost
      yesterday






    • 8




      Clarification (you might already understand this, but it is worth clarifying): pulsars don't "fire". They constantly emit their beam of light as long as the neutron star is accreting matter. The reason why they "blink" from our perspective is because the beam of light is focused (as you mention) and therefore only crosses our path once per rotation - not because it turns on and off. It's always on. The "period" is determined by its rotation rate, and doesn't have to be fast.
      – conman
      yesterday
















    42














    A wobbling pulsar will do the trick.



    Pulsars emit a lot of energy in narrow beams that come from their poles. The slowest ones flash every few seconds; make its tilt wobble so that it is not pointing at the planet all the time. In addition, wobbling causes the pulsar to shoot at different points of the planet's orbit through time. The planet is hit when the pulsar's beams' path just happens to be passing by the planet.



    If the pulsar is flashing every few milliseconds (as is normal for them), it will seem like a continuous beam for observers.



    Finally, to make the beam small enough that it doesn't cover the entire planet and more, justify it with lensing from nearby nebulas, the planet's atmosphere, and maybe a black hole between the pulsar and the planet.






    share|improve this answer



















    • 2




      Gah, I can't believe I didn't think of this. You got my +1.
      – Gryphon
      yesterday






    • 6




      +1, even easier if it CAN cover the whole planet, which I think might work for the story.
      – Devsman
      yesterday






    • 1




      Pulsars are stars that have previously exploded as supernovae: explaining life on a planet after a supernova is either a problem or interesting, depending on how you look at it.
      – Phil Frost
      yesterday






    • 3




      @Renan Meaning the pulsar isn't the center of the system but just making a pass nearby? Yeah, that could work. en.wikipedia.org/wiki/Barnard%27s_Star#/media/…
      – Phil Frost
      yesterday






    • 8




      Clarification (you might already understand this, but it is worth clarifying): pulsars don't "fire". They constantly emit their beam of light as long as the neutron star is accreting matter. The reason why they "blink" from our perspective is because the beam of light is focused (as you mention) and therefore only crosses our path once per rotation - not because it turns on and off. It's always on. The "period" is determined by its rotation rate, and doesn't have to be fast.
      – conman
      yesterday














    42












    42








    42






    A wobbling pulsar will do the trick.



    Pulsars emit a lot of energy in narrow beams that come from their poles. The slowest ones flash every few seconds; make its tilt wobble so that it is not pointing at the planet all the time. In addition, wobbling causes the pulsar to shoot at different points of the planet's orbit through time. The planet is hit when the pulsar's beams' path just happens to be passing by the planet.



    If the pulsar is flashing every few milliseconds (as is normal for them), it will seem like a continuous beam for observers.



    Finally, to make the beam small enough that it doesn't cover the entire planet and more, justify it with lensing from nearby nebulas, the planet's atmosphere, and maybe a black hole between the pulsar and the planet.






    share|improve this answer














    A wobbling pulsar will do the trick.



    Pulsars emit a lot of energy in narrow beams that come from their poles. The slowest ones flash every few seconds; make its tilt wobble so that it is not pointing at the planet all the time. In addition, wobbling causes the pulsar to shoot at different points of the planet's orbit through time. The planet is hit when the pulsar's beams' path just happens to be passing by the planet.



    If the pulsar is flashing every few milliseconds (as is normal for them), it will seem like a continuous beam for observers.



    Finally, to make the beam small enough that it doesn't cover the entire planet and more, justify it with lensing from nearby nebulas, the planet's atmosphere, and maybe a black hole between the pulsar and the planet.







    share|improve this answer














    share|improve this answer



    share|improve this answer








    edited 20 hours ago

























    answered yesterday









    RenanRenan

    44.2k11100224




    44.2k11100224








    • 2




      Gah, I can't believe I didn't think of this. You got my +1.
      – Gryphon
      yesterday






    • 6




      +1, even easier if it CAN cover the whole planet, which I think might work for the story.
      – Devsman
      yesterday






    • 1




      Pulsars are stars that have previously exploded as supernovae: explaining life on a planet after a supernova is either a problem or interesting, depending on how you look at it.
      – Phil Frost
      yesterday






    • 3




      @Renan Meaning the pulsar isn't the center of the system but just making a pass nearby? Yeah, that could work. en.wikipedia.org/wiki/Barnard%27s_Star#/media/…
      – Phil Frost
      yesterday






    • 8




      Clarification (you might already understand this, but it is worth clarifying): pulsars don't "fire". They constantly emit their beam of light as long as the neutron star is accreting matter. The reason why they "blink" from our perspective is because the beam of light is focused (as you mention) and therefore only crosses our path once per rotation - not because it turns on and off. It's always on. The "period" is determined by its rotation rate, and doesn't have to be fast.
      – conman
      yesterday














    • 2




      Gah, I can't believe I didn't think of this. You got my +1.
      – Gryphon
      yesterday






    • 6




      +1, even easier if it CAN cover the whole planet, which I think might work for the story.
      – Devsman
      yesterday






    • 1




      Pulsars are stars that have previously exploded as supernovae: explaining life on a planet after a supernova is either a problem or interesting, depending on how you look at it.
      – Phil Frost
      yesterday






    • 3




      @Renan Meaning the pulsar isn't the center of the system but just making a pass nearby? Yeah, that could work. en.wikipedia.org/wiki/Barnard%27s_Star#/media/…
      – Phil Frost
      yesterday






    • 8




      Clarification (you might already understand this, but it is worth clarifying): pulsars don't "fire". They constantly emit their beam of light as long as the neutron star is accreting matter. The reason why they "blink" from our perspective is because the beam of light is focused (as you mention) and therefore only crosses our path once per rotation - not because it turns on and off. It's always on. The "period" is determined by its rotation rate, and doesn't have to be fast.
      – conman
      yesterday








    2




    2




    Gah, I can't believe I didn't think of this. You got my +1.
    – Gryphon
    yesterday




    Gah, I can't believe I didn't think of this. You got my +1.
    – Gryphon
    yesterday




    6




    6




    +1, even easier if it CAN cover the whole planet, which I think might work for the story.
    – Devsman
    yesterday




    +1, even easier if it CAN cover the whole planet, which I think might work for the story.
    – Devsman
    yesterday




    1




    1




    Pulsars are stars that have previously exploded as supernovae: explaining life on a planet after a supernova is either a problem or interesting, depending on how you look at it.
    – Phil Frost
    yesterday




    Pulsars are stars that have previously exploded as supernovae: explaining life on a planet after a supernova is either a problem or interesting, depending on how you look at it.
    – Phil Frost
    yesterday




    3




    3




    @Renan Meaning the pulsar isn't the center of the system but just making a pass nearby? Yeah, that could work. en.wikipedia.org/wiki/Barnard%27s_Star#/media/…
    – Phil Frost
    yesterday




    @Renan Meaning the pulsar isn't the center of the system but just making a pass nearby? Yeah, that could work. en.wikipedia.org/wiki/Barnard%27s_Star#/media/…
    – Phil Frost
    yesterday




    8




    8




    Clarification (you might already understand this, but it is worth clarifying): pulsars don't "fire". They constantly emit their beam of light as long as the neutron star is accreting matter. The reason why they "blink" from our perspective is because the beam of light is focused (as you mention) and therefore only crosses our path once per rotation - not because it turns on and off. It's always on. The "period" is determined by its rotation rate, and doesn't have to be fast.
    – conman
    yesterday




    Clarification (you might already understand this, but it is worth clarifying): pulsars don't "fire". They constantly emit their beam of light as long as the neutron star is accreting matter. The reason why they "blink" from our perspective is because the beam of light is focused (as you mention) and therefore only crosses our path once per rotation - not because it turns on and off. It's always on. The "period" is determined by its rotation rate, and doesn't have to be fast.
    – conman
    yesterday











    21














    The remains of an ancient Dyson swarm



    Not quite natural, but mostly non-technological. If a prior civilization had constructed a Dyson swarm around the system's star, the light coming from the star might be heavily occluded. Assume the sun is >10X hotter than ours (or the planet is much closer in), and there are enough collector bodies in the swarm to block some 90% of the sun's light at any time. If they are close enough in toward the sun, there will be enough diffraction around each collector that they wouldn't cast visible shadows, and could only be observed by direct observation of the sun, which requires a minimum level of technology to avoid blinding yourself.



    The spotlight effect would occur when resonances in the orbital periods of the different bodies in the swarm cause gaps in coverage. The creating civilization could have arranged this purposefully to provide sunlight to further flung planets/stations, or be coincidental. The apparent brightness would grow gradually as more pieces of the swarm leave the "hole" in the field, so the warning sign would just be a rapid but gradual increase in brightness.



    The spotlight color would probably be the same as the normal sunlight. However, if the star is very hot, heading toward blue spectrum, the swarm might occlude the blue/UV portion of the spectrum more and let redder light through (imagine if each collector is a giant solar array panel with no backing, e.g. microns of silicon). At the very least they would radiate heat in the infrared. If natives to the planet are used to these conditions, that might be their normal "white" light.






    share|improve this answer





















    • Perhaps the gaps in coverage are caused by swarm members that have failed and fell out of orbit (shot down by a solar flare, by resonance with a heavy planet, collision with an in-falling comet...).
      – John Dvorak
      yesterday
















    21














    The remains of an ancient Dyson swarm



    Not quite natural, but mostly non-technological. If a prior civilization had constructed a Dyson swarm around the system's star, the light coming from the star might be heavily occluded. Assume the sun is >10X hotter than ours (or the planet is much closer in), and there are enough collector bodies in the swarm to block some 90% of the sun's light at any time. If they are close enough in toward the sun, there will be enough diffraction around each collector that they wouldn't cast visible shadows, and could only be observed by direct observation of the sun, which requires a minimum level of technology to avoid blinding yourself.



    The spotlight effect would occur when resonances in the orbital periods of the different bodies in the swarm cause gaps in coverage. The creating civilization could have arranged this purposefully to provide sunlight to further flung planets/stations, or be coincidental. The apparent brightness would grow gradually as more pieces of the swarm leave the "hole" in the field, so the warning sign would just be a rapid but gradual increase in brightness.



    The spotlight color would probably be the same as the normal sunlight. However, if the star is very hot, heading toward blue spectrum, the swarm might occlude the blue/UV portion of the spectrum more and let redder light through (imagine if each collector is a giant solar array panel with no backing, e.g. microns of silicon). At the very least they would radiate heat in the infrared. If natives to the planet are used to these conditions, that might be their normal "white" light.






    share|improve this answer





















    • Perhaps the gaps in coverage are caused by swarm members that have failed and fell out of orbit (shot down by a solar flare, by resonance with a heavy planet, collision with an in-falling comet...).
      – John Dvorak
      yesterday














    21












    21








    21






    The remains of an ancient Dyson swarm



    Not quite natural, but mostly non-technological. If a prior civilization had constructed a Dyson swarm around the system's star, the light coming from the star might be heavily occluded. Assume the sun is >10X hotter than ours (or the planet is much closer in), and there are enough collector bodies in the swarm to block some 90% of the sun's light at any time. If they are close enough in toward the sun, there will be enough diffraction around each collector that they wouldn't cast visible shadows, and could only be observed by direct observation of the sun, which requires a minimum level of technology to avoid blinding yourself.



    The spotlight effect would occur when resonances in the orbital periods of the different bodies in the swarm cause gaps in coverage. The creating civilization could have arranged this purposefully to provide sunlight to further flung planets/stations, or be coincidental. The apparent brightness would grow gradually as more pieces of the swarm leave the "hole" in the field, so the warning sign would just be a rapid but gradual increase in brightness.



    The spotlight color would probably be the same as the normal sunlight. However, if the star is very hot, heading toward blue spectrum, the swarm might occlude the blue/UV portion of the spectrum more and let redder light through (imagine if each collector is a giant solar array panel with no backing, e.g. microns of silicon). At the very least they would radiate heat in the infrared. If natives to the planet are used to these conditions, that might be their normal "white" light.






    share|improve this answer












    The remains of an ancient Dyson swarm



    Not quite natural, but mostly non-technological. If a prior civilization had constructed a Dyson swarm around the system's star, the light coming from the star might be heavily occluded. Assume the sun is >10X hotter than ours (or the planet is much closer in), and there are enough collector bodies in the swarm to block some 90% of the sun's light at any time. If they are close enough in toward the sun, there will be enough diffraction around each collector that they wouldn't cast visible shadows, and could only be observed by direct observation of the sun, which requires a minimum level of technology to avoid blinding yourself.



    The spotlight effect would occur when resonances in the orbital periods of the different bodies in the swarm cause gaps in coverage. The creating civilization could have arranged this purposefully to provide sunlight to further flung planets/stations, or be coincidental. The apparent brightness would grow gradually as more pieces of the swarm leave the "hole" in the field, so the warning sign would just be a rapid but gradual increase in brightness.



    The spotlight color would probably be the same as the normal sunlight. However, if the star is very hot, heading toward blue spectrum, the swarm might occlude the blue/UV portion of the spectrum more and let redder light through (imagine if each collector is a giant solar array panel with no backing, e.g. microns of silicon). At the very least they would radiate heat in the infrared. If natives to the planet are used to these conditions, that might be their normal "white" light.







    share|improve this answer












    share|improve this answer



    share|improve this answer










    answered yesterday









    thegreatemuthegreatemu

    1,150210




    1,150210












    • Perhaps the gaps in coverage are caused by swarm members that have failed and fell out of orbit (shot down by a solar flare, by resonance with a heavy planet, collision with an in-falling comet...).
      – John Dvorak
      yesterday


















    • Perhaps the gaps in coverage are caused by swarm members that have failed and fell out of orbit (shot down by a solar flare, by resonance with a heavy planet, collision with an in-falling comet...).
      – John Dvorak
      yesterday
















    Perhaps the gaps in coverage are caused by swarm members that have failed and fell out of orbit (shot down by a solar flare, by resonance with a heavy planet, collision with an in-falling comet...).
    – John Dvorak
    yesterday




    Perhaps the gaps in coverage are caused by swarm members that have failed and fell out of orbit (shot down by a solar flare, by resonance with a heavy planet, collision with an in-falling comet...).
    – John Dvorak
    yesterday











    9














    Any sort of passive light-focusing (with lenses, mirrors, etc) scheme is unlikely do more than to make slightly warm spots. The fundamental reason has to do with the conservation of etendue, and you can read more about it at Would a Moon made of water pose a threat to Earth during eclipses?



    As such, if you want the spotlight to come from a moon, the moon would either need some kind of power source (which starts to sound like "giant lasers") or would need some natural mechanism to eject jets of energy or matter. As far as I know, all kinds of astrophysical jets would require something much more massive than a moon, so this seems like a dead-end.



    I think the most feasible explanation is a planet which is ordinarily protected by its atmosphere and/or magnetosphere, but on occasion the weather aligns such that the protection is lost in an area. We to experience this to a small extent on earth: both the sun and earth have magnetic fields that vary over time. One trouble is if the earth's magnetosphere were periodically penetrated by the solar wind, the atmosphere would be stripped away. Though it could take a very long time -- perhaps it is interesting for your story to have a "dying planet".






    share|improve this answer





















    • +1. If it enables the scenario, a dying planet is perfectly fine. It doesn't matter what happens to the planet after the story ends. :P
      – Devsman
      yesterday
















    9














    Any sort of passive light-focusing (with lenses, mirrors, etc) scheme is unlikely do more than to make slightly warm spots. The fundamental reason has to do with the conservation of etendue, and you can read more about it at Would a Moon made of water pose a threat to Earth during eclipses?



    As such, if you want the spotlight to come from a moon, the moon would either need some kind of power source (which starts to sound like "giant lasers") or would need some natural mechanism to eject jets of energy or matter. As far as I know, all kinds of astrophysical jets would require something much more massive than a moon, so this seems like a dead-end.



    I think the most feasible explanation is a planet which is ordinarily protected by its atmosphere and/or magnetosphere, but on occasion the weather aligns such that the protection is lost in an area. We to experience this to a small extent on earth: both the sun and earth have magnetic fields that vary over time. One trouble is if the earth's magnetosphere were periodically penetrated by the solar wind, the atmosphere would be stripped away. Though it could take a very long time -- perhaps it is interesting for your story to have a "dying planet".






    share|improve this answer





















    • +1. If it enables the scenario, a dying planet is perfectly fine. It doesn't matter what happens to the planet after the story ends. :P
      – Devsman
      yesterday














    9












    9








    9






    Any sort of passive light-focusing (with lenses, mirrors, etc) scheme is unlikely do more than to make slightly warm spots. The fundamental reason has to do with the conservation of etendue, and you can read more about it at Would a Moon made of water pose a threat to Earth during eclipses?



    As such, if you want the spotlight to come from a moon, the moon would either need some kind of power source (which starts to sound like "giant lasers") or would need some natural mechanism to eject jets of energy or matter. As far as I know, all kinds of astrophysical jets would require something much more massive than a moon, so this seems like a dead-end.



    I think the most feasible explanation is a planet which is ordinarily protected by its atmosphere and/or magnetosphere, but on occasion the weather aligns such that the protection is lost in an area. We to experience this to a small extent on earth: both the sun and earth have magnetic fields that vary over time. One trouble is if the earth's magnetosphere were periodically penetrated by the solar wind, the atmosphere would be stripped away. Though it could take a very long time -- perhaps it is interesting for your story to have a "dying planet".






    share|improve this answer












    Any sort of passive light-focusing (with lenses, mirrors, etc) scheme is unlikely do more than to make slightly warm spots. The fundamental reason has to do with the conservation of etendue, and you can read more about it at Would a Moon made of water pose a threat to Earth during eclipses?



    As such, if you want the spotlight to come from a moon, the moon would either need some kind of power source (which starts to sound like "giant lasers") or would need some natural mechanism to eject jets of energy or matter. As far as I know, all kinds of astrophysical jets would require something much more massive than a moon, so this seems like a dead-end.



    I think the most feasible explanation is a planet which is ordinarily protected by its atmosphere and/or magnetosphere, but on occasion the weather aligns such that the protection is lost in an area. We to experience this to a small extent on earth: both the sun and earth have magnetic fields that vary over time. One trouble is if the earth's magnetosphere were periodically penetrated by the solar wind, the atmosphere would be stripped away. Though it could take a very long time -- perhaps it is interesting for your story to have a "dying planet".







    share|improve this answer












    share|improve this answer



    share|improve this answer










    answered yesterday









    Phil FrostPhil Frost

    1,946159




    1,946159












    • +1. If it enables the scenario, a dying planet is perfectly fine. It doesn't matter what happens to the planet after the story ends. :P
      – Devsman
      yesterday


















    • +1. If it enables the scenario, a dying planet is perfectly fine. It doesn't matter what happens to the planet after the story ends. :P
      – Devsman
      yesterday
















    +1. If it enables the scenario, a dying planet is perfectly fine. It doesn't matter what happens to the planet after the story ends. :P
    – Devsman
    yesterday




    +1. If it enables the scenario, a dying planet is perfectly fine. It doesn't matter what happens to the planet after the story ends. :P
    – Devsman
    yesterday











    4














    Not entirely sure about the feasibility of this, but it's an idea I had when I read your question. I wonder if something like this would be possible through Gravitational Lensing. Essentially, this is when black holes (with enormous gravitational pulls) bend light around them, causing telescopic effects. I've linked the Wikipedia page for gravitational lensing as well as an article from Space.com below which you could read up on to give you a better idea of how it all works.



    My idea though is that what if, just outside the edge of what can be seen from the planet, there's a system of black holes which pull light in such a manner that it's focussed into a thin beam, which cuts across the galaxy and occasionally burns its way across your planet? This would explain the huge intensity of the light as well as allowing a 'natural' explanation for how it's focussed so tightly.



    Having it at such a distance would also mean that the appearance of the beam of light can't be predicted, as the people on the planet don't have the technology to either see that far into space, or understand what they're seeing. Besides, at such a distance that the black holes don't mess with the solar system's structure, the light would take very long to reach the planet. So when the beam lines up, its effects are only seen on the planet later (how much later depends on the distance. Centuries, or even millennia maybe).



    As for the colour of the beam, we can assume the source of light is moving away from the black holes and the planet, which would cause Redshift. This would make a white light source look red to the observer. Frequency wise, we could assume that other objects in space (planets, dust clouds etc) often block the light from hitting our planet, but occasionally it slips through the gaps (like when you see the sun for a couple of seconds through a clearing in the clouds before it gets blocked again).



    For non-animals to survive, perhaps they've evolved to feed on the huge light intensity and maybe even need it every few days to live? Or (depending on the history of the 'humans' on your planet) maybe everything else is evolved to survive the intense light to an extent, while humans aren't. Perhaps this is similar to how we have to wear clothes - we can't handle Earth's natural climates without external help. Maybe those caught outside of their radiation booths are killed rapidly, while those who stay inside are fine?



    There's a lot of ways you can go with this idea and I think the rest is up to you. I've included a few links at the bottom you might be interested in.



    Further Reading:



    Gravitational lensing:



    https://www.space.com/39999-how-gravitational-lenses-work.html



    https://en.wikipedia.org/wiki/Gravitational_lens



    Red and blue shift:



    https://en.wikipedia.org/wiki/Redshift



    https://www.space.com/25732-redshift-blueshift.html






    share|improve this answer





















    • +1 for redshift
      – Devsman
      yesterday






    • 8




      Sadly, the conservation of etendue makes this system of black hole lenses unworkable. If you're concentrating the light to a smaller output area, then it must also be distributed across a larger angle as it leaves the lens. XKCD's Randall Munroe did an excellent post on this topic: what-if.xkcd.com/145
      – Dubukay
      yesterday






    • 1




      @Dubukay wow, never heard of that conservation law before. Thanks for bringing it to my attention! Love learning stuff on here haha
      – user43712
      yesterday






    • 4




      However, if what we're going for is suspension-of-disbelief, make it a mini black hole in an inner orbit of the same system. Each time that BH passes between your unfortunate planet and the sun, lensing burns a scar right across the planet. If there's enough sunlight to start a fire with a magnifying glass, there's enough to do it through gravitational lensing (note that most of the planet will be nearly dark due to the lensing effect).
      – JBH
      yesterday






    • 5




      Also, keep in mind that the mini black hole could also be one half of a binary star, in which case the orbits are preserved and the effect is the same, if a bit more complicated to calculate.
      – JBH
      yesterday
















    4














    Not entirely sure about the feasibility of this, but it's an idea I had when I read your question. I wonder if something like this would be possible through Gravitational Lensing. Essentially, this is when black holes (with enormous gravitational pulls) bend light around them, causing telescopic effects. I've linked the Wikipedia page for gravitational lensing as well as an article from Space.com below which you could read up on to give you a better idea of how it all works.



    My idea though is that what if, just outside the edge of what can be seen from the planet, there's a system of black holes which pull light in such a manner that it's focussed into a thin beam, which cuts across the galaxy and occasionally burns its way across your planet? This would explain the huge intensity of the light as well as allowing a 'natural' explanation for how it's focussed so tightly.



    Having it at such a distance would also mean that the appearance of the beam of light can't be predicted, as the people on the planet don't have the technology to either see that far into space, or understand what they're seeing. Besides, at such a distance that the black holes don't mess with the solar system's structure, the light would take very long to reach the planet. So when the beam lines up, its effects are only seen on the planet later (how much later depends on the distance. Centuries, or even millennia maybe).



    As for the colour of the beam, we can assume the source of light is moving away from the black holes and the planet, which would cause Redshift. This would make a white light source look red to the observer. Frequency wise, we could assume that other objects in space (planets, dust clouds etc) often block the light from hitting our planet, but occasionally it slips through the gaps (like when you see the sun for a couple of seconds through a clearing in the clouds before it gets blocked again).



    For non-animals to survive, perhaps they've evolved to feed on the huge light intensity and maybe even need it every few days to live? Or (depending on the history of the 'humans' on your planet) maybe everything else is evolved to survive the intense light to an extent, while humans aren't. Perhaps this is similar to how we have to wear clothes - we can't handle Earth's natural climates without external help. Maybe those caught outside of their radiation booths are killed rapidly, while those who stay inside are fine?



    There's a lot of ways you can go with this idea and I think the rest is up to you. I've included a few links at the bottom you might be interested in.



    Further Reading:



    Gravitational lensing:



    https://www.space.com/39999-how-gravitational-lenses-work.html



    https://en.wikipedia.org/wiki/Gravitational_lens



    Red and blue shift:



    https://en.wikipedia.org/wiki/Redshift



    https://www.space.com/25732-redshift-blueshift.html






    share|improve this answer





















    • +1 for redshift
      – Devsman
      yesterday






    • 8




      Sadly, the conservation of etendue makes this system of black hole lenses unworkable. If you're concentrating the light to a smaller output area, then it must also be distributed across a larger angle as it leaves the lens. XKCD's Randall Munroe did an excellent post on this topic: what-if.xkcd.com/145
      – Dubukay
      yesterday






    • 1




      @Dubukay wow, never heard of that conservation law before. Thanks for bringing it to my attention! Love learning stuff on here haha
      – user43712
      yesterday






    • 4




      However, if what we're going for is suspension-of-disbelief, make it a mini black hole in an inner orbit of the same system. Each time that BH passes between your unfortunate planet and the sun, lensing burns a scar right across the planet. If there's enough sunlight to start a fire with a magnifying glass, there's enough to do it through gravitational lensing (note that most of the planet will be nearly dark due to the lensing effect).
      – JBH
      yesterday






    • 5




      Also, keep in mind that the mini black hole could also be one half of a binary star, in which case the orbits are preserved and the effect is the same, if a bit more complicated to calculate.
      – JBH
      yesterday














    4












    4








    4






    Not entirely sure about the feasibility of this, but it's an idea I had when I read your question. I wonder if something like this would be possible through Gravitational Lensing. Essentially, this is when black holes (with enormous gravitational pulls) bend light around them, causing telescopic effects. I've linked the Wikipedia page for gravitational lensing as well as an article from Space.com below which you could read up on to give you a better idea of how it all works.



    My idea though is that what if, just outside the edge of what can be seen from the planet, there's a system of black holes which pull light in such a manner that it's focussed into a thin beam, which cuts across the galaxy and occasionally burns its way across your planet? This would explain the huge intensity of the light as well as allowing a 'natural' explanation for how it's focussed so tightly.



    Having it at such a distance would also mean that the appearance of the beam of light can't be predicted, as the people on the planet don't have the technology to either see that far into space, or understand what they're seeing. Besides, at such a distance that the black holes don't mess with the solar system's structure, the light would take very long to reach the planet. So when the beam lines up, its effects are only seen on the planet later (how much later depends on the distance. Centuries, or even millennia maybe).



    As for the colour of the beam, we can assume the source of light is moving away from the black holes and the planet, which would cause Redshift. This would make a white light source look red to the observer. Frequency wise, we could assume that other objects in space (planets, dust clouds etc) often block the light from hitting our planet, but occasionally it slips through the gaps (like when you see the sun for a couple of seconds through a clearing in the clouds before it gets blocked again).



    For non-animals to survive, perhaps they've evolved to feed on the huge light intensity and maybe even need it every few days to live? Or (depending on the history of the 'humans' on your planet) maybe everything else is evolved to survive the intense light to an extent, while humans aren't. Perhaps this is similar to how we have to wear clothes - we can't handle Earth's natural climates without external help. Maybe those caught outside of their radiation booths are killed rapidly, while those who stay inside are fine?



    There's a lot of ways you can go with this idea and I think the rest is up to you. I've included a few links at the bottom you might be interested in.



    Further Reading:



    Gravitational lensing:



    https://www.space.com/39999-how-gravitational-lenses-work.html



    https://en.wikipedia.org/wiki/Gravitational_lens



    Red and blue shift:



    https://en.wikipedia.org/wiki/Redshift



    https://www.space.com/25732-redshift-blueshift.html






    share|improve this answer












    Not entirely sure about the feasibility of this, but it's an idea I had when I read your question. I wonder if something like this would be possible through Gravitational Lensing. Essentially, this is when black holes (with enormous gravitational pulls) bend light around them, causing telescopic effects. I've linked the Wikipedia page for gravitational lensing as well as an article from Space.com below which you could read up on to give you a better idea of how it all works.



    My idea though is that what if, just outside the edge of what can be seen from the planet, there's a system of black holes which pull light in such a manner that it's focussed into a thin beam, which cuts across the galaxy and occasionally burns its way across your planet? This would explain the huge intensity of the light as well as allowing a 'natural' explanation for how it's focussed so tightly.



    Having it at such a distance would also mean that the appearance of the beam of light can't be predicted, as the people on the planet don't have the technology to either see that far into space, or understand what they're seeing. Besides, at such a distance that the black holes don't mess with the solar system's structure, the light would take very long to reach the planet. So when the beam lines up, its effects are only seen on the planet later (how much later depends on the distance. Centuries, or even millennia maybe).



    As for the colour of the beam, we can assume the source of light is moving away from the black holes and the planet, which would cause Redshift. This would make a white light source look red to the observer. Frequency wise, we could assume that other objects in space (planets, dust clouds etc) often block the light from hitting our planet, but occasionally it slips through the gaps (like when you see the sun for a couple of seconds through a clearing in the clouds before it gets blocked again).



    For non-animals to survive, perhaps they've evolved to feed on the huge light intensity and maybe even need it every few days to live? Or (depending on the history of the 'humans' on your planet) maybe everything else is evolved to survive the intense light to an extent, while humans aren't. Perhaps this is similar to how we have to wear clothes - we can't handle Earth's natural climates without external help. Maybe those caught outside of their radiation booths are killed rapidly, while those who stay inside are fine?



    There's a lot of ways you can go with this idea and I think the rest is up to you. I've included a few links at the bottom you might be interested in.



    Further Reading:



    Gravitational lensing:



    https://www.space.com/39999-how-gravitational-lenses-work.html



    https://en.wikipedia.org/wiki/Gravitational_lens



    Red and blue shift:



    https://en.wikipedia.org/wiki/Redshift



    https://www.space.com/25732-redshift-blueshift.html







    share|improve this answer












    share|improve this answer



    share|improve this answer










    answered yesterday









    user43712user43712

    1192




    1192












    • +1 for redshift
      – Devsman
      yesterday






    • 8




      Sadly, the conservation of etendue makes this system of black hole lenses unworkable. If you're concentrating the light to a smaller output area, then it must also be distributed across a larger angle as it leaves the lens. XKCD's Randall Munroe did an excellent post on this topic: what-if.xkcd.com/145
      – Dubukay
      yesterday






    • 1




      @Dubukay wow, never heard of that conservation law before. Thanks for bringing it to my attention! Love learning stuff on here haha
      – user43712
      yesterday






    • 4




      However, if what we're going for is suspension-of-disbelief, make it a mini black hole in an inner orbit of the same system. Each time that BH passes between your unfortunate planet and the sun, lensing burns a scar right across the planet. If there's enough sunlight to start a fire with a magnifying glass, there's enough to do it through gravitational lensing (note that most of the planet will be nearly dark due to the lensing effect).
      – JBH
      yesterday






    • 5




      Also, keep in mind that the mini black hole could also be one half of a binary star, in which case the orbits are preserved and the effect is the same, if a bit more complicated to calculate.
      – JBH
      yesterday


















    • +1 for redshift
      – Devsman
      yesterday






    • 8




      Sadly, the conservation of etendue makes this system of black hole lenses unworkable. If you're concentrating the light to a smaller output area, then it must also be distributed across a larger angle as it leaves the lens. XKCD's Randall Munroe did an excellent post on this topic: what-if.xkcd.com/145
      – Dubukay
      yesterday






    • 1




      @Dubukay wow, never heard of that conservation law before. Thanks for bringing it to my attention! Love learning stuff on here haha
      – user43712
      yesterday






    • 4




      However, if what we're going for is suspension-of-disbelief, make it a mini black hole in an inner orbit of the same system. Each time that BH passes between your unfortunate planet and the sun, lensing burns a scar right across the planet. If there's enough sunlight to start a fire with a magnifying glass, there's enough to do it through gravitational lensing (note that most of the planet will be nearly dark due to the lensing effect).
      – JBH
      yesterday






    • 5




      Also, keep in mind that the mini black hole could also be one half of a binary star, in which case the orbits are preserved and the effect is the same, if a bit more complicated to calculate.
      – JBH
      yesterday
















    +1 for redshift
    – Devsman
    yesterday




    +1 for redshift
    – Devsman
    yesterday




    8




    8




    Sadly, the conservation of etendue makes this system of black hole lenses unworkable. If you're concentrating the light to a smaller output area, then it must also be distributed across a larger angle as it leaves the lens. XKCD's Randall Munroe did an excellent post on this topic: what-if.xkcd.com/145
    – Dubukay
    yesterday




    Sadly, the conservation of etendue makes this system of black hole lenses unworkable. If you're concentrating the light to a smaller output area, then it must also be distributed across a larger angle as it leaves the lens. XKCD's Randall Munroe did an excellent post on this topic: what-if.xkcd.com/145
    – Dubukay
    yesterday




    1




    1




    @Dubukay wow, never heard of that conservation law before. Thanks for bringing it to my attention! Love learning stuff on here haha
    – user43712
    yesterday




    @Dubukay wow, never heard of that conservation law before. Thanks for bringing it to my attention! Love learning stuff on here haha
    – user43712
    yesterday




    4




    4




    However, if what we're going for is suspension-of-disbelief, make it a mini black hole in an inner orbit of the same system. Each time that BH passes between your unfortunate planet and the sun, lensing burns a scar right across the planet. If there's enough sunlight to start a fire with a magnifying glass, there's enough to do it through gravitational lensing (note that most of the planet will be nearly dark due to the lensing effect).
    – JBH
    yesterday




    However, if what we're going for is suspension-of-disbelief, make it a mini black hole in an inner orbit of the same system. Each time that BH passes between your unfortunate planet and the sun, lensing burns a scar right across the planet. If there's enough sunlight to start a fire with a magnifying glass, there's enough to do it through gravitational lensing (note that most of the planet will be nearly dark due to the lensing effect).
    – JBH
    yesterday




    5




    5




    Also, keep in mind that the mini black hole could also be one half of a binary star, in which case the orbits are preserved and the effect is the same, if a bit more complicated to calculate.
    – JBH
    yesterday




    Also, keep in mind that the mini black hole could also be one half of a binary star, in which case the orbits are preserved and the effect is the same, if a bit more complicated to calculate.
    – JBH
    yesterday











    2














    Ozone holes (https://en.wikipedia.org/wiki/Ozone_depletion) already cause real world health problems.



    I think you could combine ozone holes and the loss of other atmospheric protection with the a coinciding local weakening of the magnetosphere. In the presence of a very energetic "sun", the results might be "deadly".



    "Deadly" as in it might be immediately painful and certainly cancerous over time. Not deadly in the Vin Diesel kind of way you might be looking for though.






    share|improve this answer





















    • A magnetosphere has no effect on electromagnetic radiation, and a breathable atmosphere would protect against solar wind.
      – Christopher James Huff
      yesterday
















    2














    Ozone holes (https://en.wikipedia.org/wiki/Ozone_depletion) already cause real world health problems.



    I think you could combine ozone holes and the loss of other atmospheric protection with the a coinciding local weakening of the magnetosphere. In the presence of a very energetic "sun", the results might be "deadly".



    "Deadly" as in it might be immediately painful and certainly cancerous over time. Not deadly in the Vin Diesel kind of way you might be looking for though.






    share|improve this answer





















    • A magnetosphere has no effect on electromagnetic radiation, and a breathable atmosphere would protect against solar wind.
      – Christopher James Huff
      yesterday














    2












    2








    2






    Ozone holes (https://en.wikipedia.org/wiki/Ozone_depletion) already cause real world health problems.



    I think you could combine ozone holes and the loss of other atmospheric protection with the a coinciding local weakening of the magnetosphere. In the presence of a very energetic "sun", the results might be "deadly".



    "Deadly" as in it might be immediately painful and certainly cancerous over time. Not deadly in the Vin Diesel kind of way you might be looking for though.






    share|improve this answer












    Ozone holes (https://en.wikipedia.org/wiki/Ozone_depletion) already cause real world health problems.



    I think you could combine ozone holes and the loss of other atmospheric protection with the a coinciding local weakening of the magnetosphere. In the presence of a very energetic "sun", the results might be "deadly".



    "Deadly" as in it might be immediately painful and certainly cancerous over time. Not deadly in the Vin Diesel kind of way you might be looking for though.







    share|improve this answer












    share|improve this answer



    share|improve this answer










    answered yesterday









    JonSGJonSG

    45125




    45125












    • A magnetosphere has no effect on electromagnetic radiation, and a breathable atmosphere would protect against solar wind.
      – Christopher James Huff
      yesterday


















    • A magnetosphere has no effect on electromagnetic radiation, and a breathable atmosphere would protect against solar wind.
      – Christopher James Huff
      yesterday
















    A magnetosphere has no effect on electromagnetic radiation, and a breathable atmosphere would protect against solar wind.
    – Christopher James Huff
    yesterday




    A magnetosphere has no effect on electromagnetic radiation, and a breathable atmosphere would protect against solar wind.
    – Christopher James Huff
    yesterday











    2














    Let's have some cosmic fun.



    Say that your planet orbits inside of an expanding red giant star.



    Yep.



    Now, say that someone or something decided to construct a shell around your world, perhaps before it had been engulfed by your host star as it evolved from the main sequence. This shell may be layers upon layers of orbital rings.



    Now, say that this shell--made of some impossible material, probably--has some degree of translucency to it and the ability to change this translucency. You can have one hemisphere of the shell totally opaque to simulate night, with perhaps little points of translucency to simulate stars, and the other hemisphere translucent to simulate daylight (much less a simulation this point, because actual starlight would be entering).



    Finally, let this shell have some circular region of total transparency which sweeps the planet, perhaps moving across the shell faster than the day-night cycle to give the folks on all sides, day or night, some of the action. You can think of the shell as being comprised of a bunch of little windows if you'd like, like pixels on a screen, that can be tinted and whatnot and can be made to filter out the unnecessary or harmful radiation of the star it is orbiting inside. The transparent portion would not filter these things, allowing death to beam down onto everything below. Heck, perhaps it even has little perforations or 'openings of the windows' that allow actual, high-energy particles to enter as well.






    share|improve this answer



















    • 1




      "Say that your planet orbits inside of an expanding red giant star." It would quickly deorbit due to friction.
      – Renan
      yesterday






    • 1




      @Renan Not as quickly as you'd think.
      – B.fox
      yesterday










    • @B.fox, but still very quickly on a scale of planets. A very rough approximation, pretending that the classic drag equation is valid at all speeds, is that an Earth-sized planet will come to a complete stop after about 400 years.
      – Mark
      yesterday












    • @Mark I'll have to follow up on that. I'm pretty sure the range was somewhere in the hundreds of thousands of years. The outer atmosphere of a red giant star is pretty diffuse, almost insubstantial.
      – B.fox
      22 hours ago












    • I'm not sure why this solution should depend on the planet being inside the sun. Take our own system, build such a shell around Mercury.
      – Mr Lister
      12 hours ago


















    2














    Let's have some cosmic fun.



    Say that your planet orbits inside of an expanding red giant star.



    Yep.



    Now, say that someone or something decided to construct a shell around your world, perhaps before it had been engulfed by your host star as it evolved from the main sequence. This shell may be layers upon layers of orbital rings.



    Now, say that this shell--made of some impossible material, probably--has some degree of translucency to it and the ability to change this translucency. You can have one hemisphere of the shell totally opaque to simulate night, with perhaps little points of translucency to simulate stars, and the other hemisphere translucent to simulate daylight (much less a simulation this point, because actual starlight would be entering).



    Finally, let this shell have some circular region of total transparency which sweeps the planet, perhaps moving across the shell faster than the day-night cycle to give the folks on all sides, day or night, some of the action. You can think of the shell as being comprised of a bunch of little windows if you'd like, like pixels on a screen, that can be tinted and whatnot and can be made to filter out the unnecessary or harmful radiation of the star it is orbiting inside. The transparent portion would not filter these things, allowing death to beam down onto everything below. Heck, perhaps it even has little perforations or 'openings of the windows' that allow actual, high-energy particles to enter as well.






    share|improve this answer



















    • 1




      "Say that your planet orbits inside of an expanding red giant star." It would quickly deorbit due to friction.
      – Renan
      yesterday






    • 1




      @Renan Not as quickly as you'd think.
      – B.fox
      yesterday










    • @B.fox, but still very quickly on a scale of planets. A very rough approximation, pretending that the classic drag equation is valid at all speeds, is that an Earth-sized planet will come to a complete stop after about 400 years.
      – Mark
      yesterday












    • @Mark I'll have to follow up on that. I'm pretty sure the range was somewhere in the hundreds of thousands of years. The outer atmosphere of a red giant star is pretty diffuse, almost insubstantial.
      – B.fox
      22 hours ago












    • I'm not sure why this solution should depend on the planet being inside the sun. Take our own system, build such a shell around Mercury.
      – Mr Lister
      12 hours ago
















    2












    2








    2






    Let's have some cosmic fun.



    Say that your planet orbits inside of an expanding red giant star.



    Yep.



    Now, say that someone or something decided to construct a shell around your world, perhaps before it had been engulfed by your host star as it evolved from the main sequence. This shell may be layers upon layers of orbital rings.



    Now, say that this shell--made of some impossible material, probably--has some degree of translucency to it and the ability to change this translucency. You can have one hemisphere of the shell totally opaque to simulate night, with perhaps little points of translucency to simulate stars, and the other hemisphere translucent to simulate daylight (much less a simulation this point, because actual starlight would be entering).



    Finally, let this shell have some circular region of total transparency which sweeps the planet, perhaps moving across the shell faster than the day-night cycle to give the folks on all sides, day or night, some of the action. You can think of the shell as being comprised of a bunch of little windows if you'd like, like pixels on a screen, that can be tinted and whatnot and can be made to filter out the unnecessary or harmful radiation of the star it is orbiting inside. The transparent portion would not filter these things, allowing death to beam down onto everything below. Heck, perhaps it even has little perforations or 'openings of the windows' that allow actual, high-energy particles to enter as well.






    share|improve this answer














    Let's have some cosmic fun.



    Say that your planet orbits inside of an expanding red giant star.



    Yep.



    Now, say that someone or something decided to construct a shell around your world, perhaps before it had been engulfed by your host star as it evolved from the main sequence. This shell may be layers upon layers of orbital rings.



    Now, say that this shell--made of some impossible material, probably--has some degree of translucency to it and the ability to change this translucency. You can have one hemisphere of the shell totally opaque to simulate night, with perhaps little points of translucency to simulate stars, and the other hemisphere translucent to simulate daylight (much less a simulation this point, because actual starlight would be entering).



    Finally, let this shell have some circular region of total transparency which sweeps the planet, perhaps moving across the shell faster than the day-night cycle to give the folks on all sides, day or night, some of the action. You can think of the shell as being comprised of a bunch of little windows if you'd like, like pixels on a screen, that can be tinted and whatnot and can be made to filter out the unnecessary or harmful radiation of the star it is orbiting inside. The transparent portion would not filter these things, allowing death to beam down onto everything below. Heck, perhaps it even has little perforations or 'openings of the windows' that allow actual, high-energy particles to enter as well.







    share|improve this answer














    share|improve this answer



    share|improve this answer








    edited yesterday

























    answered yesterday









    B.foxB.fox

    9491316




    9491316








    • 1




      "Say that your planet orbits inside of an expanding red giant star." It would quickly deorbit due to friction.
      – Renan
      yesterday






    • 1




      @Renan Not as quickly as you'd think.
      – B.fox
      yesterday










    • @B.fox, but still very quickly on a scale of planets. A very rough approximation, pretending that the classic drag equation is valid at all speeds, is that an Earth-sized planet will come to a complete stop after about 400 years.
      – Mark
      yesterday












    • @Mark I'll have to follow up on that. I'm pretty sure the range was somewhere in the hundreds of thousands of years. The outer atmosphere of a red giant star is pretty diffuse, almost insubstantial.
      – B.fox
      22 hours ago












    • I'm not sure why this solution should depend on the planet being inside the sun. Take our own system, build such a shell around Mercury.
      – Mr Lister
      12 hours ago
















    • 1




      "Say that your planet orbits inside of an expanding red giant star." It would quickly deorbit due to friction.
      – Renan
      yesterday






    • 1




      @Renan Not as quickly as you'd think.
      – B.fox
      yesterday










    • @B.fox, but still very quickly on a scale of planets. A very rough approximation, pretending that the classic drag equation is valid at all speeds, is that an Earth-sized planet will come to a complete stop after about 400 years.
      – Mark
      yesterday












    • @Mark I'll have to follow up on that. I'm pretty sure the range was somewhere in the hundreds of thousands of years. The outer atmosphere of a red giant star is pretty diffuse, almost insubstantial.
      – B.fox
      22 hours ago












    • I'm not sure why this solution should depend on the planet being inside the sun. Take our own system, build such a shell around Mercury.
      – Mr Lister
      12 hours ago










    1




    1




    "Say that your planet orbits inside of an expanding red giant star." It would quickly deorbit due to friction.
    – Renan
    yesterday




    "Say that your planet orbits inside of an expanding red giant star." It would quickly deorbit due to friction.
    – Renan
    yesterday




    1




    1




    @Renan Not as quickly as you'd think.
    – B.fox
    yesterday




    @Renan Not as quickly as you'd think.
    – B.fox
    yesterday












    @B.fox, but still very quickly on a scale of planets. A very rough approximation, pretending that the classic drag equation is valid at all speeds, is that an Earth-sized planet will come to a complete stop after about 400 years.
    – Mark
    yesterday






    @B.fox, but still very quickly on a scale of planets. A very rough approximation, pretending that the classic drag equation is valid at all speeds, is that an Earth-sized planet will come to a complete stop after about 400 years.
    – Mark
    yesterday














    @Mark I'll have to follow up on that. I'm pretty sure the range was somewhere in the hundreds of thousands of years. The outer atmosphere of a red giant star is pretty diffuse, almost insubstantial.
    – B.fox
    22 hours ago






    @Mark I'll have to follow up on that. I'm pretty sure the range was somewhere in the hundreds of thousands of years. The outer atmosphere of a red giant star is pretty diffuse, almost insubstantial.
    – B.fox
    22 hours ago














    I'm not sure why this solution should depend on the planet being inside the sun. Take our own system, build such a shell around Mercury.
    – Mr Lister
    12 hours ago






    I'm not sure why this solution should depend on the planet being inside the sun. Take our own system, build such a shell around Mercury.
    – Mr Lister
    12 hours ago













    1














    Constant lightning.



    Consider a Rocheworld.
    Can an atmosphere englobe a planetary ring?




    Two tidally locked planets just outside the Roche Limit can orbit each
    other and share a combined atmosphere. You would be able to fly from
    one to the other without ever leaving the atmosphere and objects
    placed at the lagrange points would be able to remain there.




    These binary planets circle around each other. At one point in their orbit, their atmospheres touch (or you could have a moon graze the atmosphere of its planet). The friction of the two atmospheres against each other produces colossal amounts of electrical charge. When the two are close enough that the atmosphere can provide a path, charge can equilibrate across.



    This takes the form of constant tremendous bolts of lightning that follow the path of the point where the two partners are closest to one another.






    share|improve this answer


























      1














      Constant lightning.



      Consider a Rocheworld.
      Can an atmosphere englobe a planetary ring?




      Two tidally locked planets just outside the Roche Limit can orbit each
      other and share a combined atmosphere. You would be able to fly from
      one to the other without ever leaving the atmosphere and objects
      placed at the lagrange points would be able to remain there.




      These binary planets circle around each other. At one point in their orbit, their atmospheres touch (or you could have a moon graze the atmosphere of its planet). The friction of the two atmospheres against each other produces colossal amounts of electrical charge. When the two are close enough that the atmosphere can provide a path, charge can equilibrate across.



      This takes the form of constant tremendous bolts of lightning that follow the path of the point where the two partners are closest to one another.






      share|improve this answer
























        1












        1








        1






        Constant lightning.



        Consider a Rocheworld.
        Can an atmosphere englobe a planetary ring?




        Two tidally locked planets just outside the Roche Limit can orbit each
        other and share a combined atmosphere. You would be able to fly from
        one to the other without ever leaving the atmosphere and objects
        placed at the lagrange points would be able to remain there.




        These binary planets circle around each other. At one point in their orbit, their atmospheres touch (or you could have a moon graze the atmosphere of its planet). The friction of the two atmospheres against each other produces colossal amounts of electrical charge. When the two are close enough that the atmosphere can provide a path, charge can equilibrate across.



        This takes the form of constant tremendous bolts of lightning that follow the path of the point where the two partners are closest to one another.






        share|improve this answer












        Constant lightning.



        Consider a Rocheworld.
        Can an atmosphere englobe a planetary ring?




        Two tidally locked planets just outside the Roche Limit can orbit each
        other and share a combined atmosphere. You would be able to fly from
        one to the other without ever leaving the atmosphere and objects
        placed at the lagrange points would be able to remain there.




        These binary planets circle around each other. At one point in their orbit, their atmospheres touch (or you could have a moon graze the atmosphere of its planet). The friction of the two atmospheres against each other produces colossal amounts of electrical charge. When the two are close enough that the atmosphere can provide a path, charge can equilibrate across.



        This takes the form of constant tremendous bolts of lightning that follow the path of the point where the two partners are closest to one another.







        share|improve this answer












        share|improve this answer



        share|improve this answer










        answered yesterday









        WillkWillk

        103k25197433




        103k25197433























            1














            I haven't seen this directly addressed, so I'll pose it as an answer:



            TL;DR: Large moon with atmosphere refracts to a "point" on your planet. See below for etendue/thermodynamics, refraction, periodicity, and "warning signs".



            A companion (moon, twin planet, or even planet as primary with your story set on a comfortable moon of a gas giant) large enough to hold a substantial atmosphere can perhaps be tuned to get the result you need.



            Devise an atmosphere for the companion body with a powerful thermal inversion somewhere that reduces some of the spreading due to typical refraction of a density-stratified lens.



            So we effectively have a ring-shaped lens, fairly narrow (edge view of the companion's atmosphere) but of very wide diameter (the companion itself), tuned to refract fairly well to a "point". The source of the light is the sun, and we will not get hotter than that. We do not need perfect point focus, but will gladly accept a central line of foci for various degrees of refraction, which generate -- you guessed it -- different colors of spotlight at different orbital distance of the large body from your planet's surface. Blue when it's close, red when it's far -- if it behaves like a proper lens-shaped lens. This would also result in color change as the effect sweeps from the edge of the home planet (farther) to the center (a bit closer). Warning signs would be similar to normal eclipses (the effect would only be observable from the very height of the eclipse). Finally, a combination of rotational and orbital planes for the three bodies involved can do wonders for making a simple periodic set of processes appear miserably non-periodic, particularly for observers located at different points on the surface of the home body.



            I'll create a graphic. But I think this thing is doable with a lot less machinery than has been proposed so far, and without violating physics to the point of ridicule.






            share|improve this answer



















            • 1




              The only reason this won't get ridiculed for violating physics is that most people don't know about conservation of etendue.
              – Mark
              yesterday










            • Mark, how so? A garden variety magnifying glass does not get as hot as the point focused upon.
              – Haakon Dahl
              7 mins ago
















            1














            I haven't seen this directly addressed, so I'll pose it as an answer:



            TL;DR: Large moon with atmosphere refracts to a "point" on your planet. See below for etendue/thermodynamics, refraction, periodicity, and "warning signs".



            A companion (moon, twin planet, or even planet as primary with your story set on a comfortable moon of a gas giant) large enough to hold a substantial atmosphere can perhaps be tuned to get the result you need.



            Devise an atmosphere for the companion body with a powerful thermal inversion somewhere that reduces some of the spreading due to typical refraction of a density-stratified lens.



            So we effectively have a ring-shaped lens, fairly narrow (edge view of the companion's atmosphere) but of very wide diameter (the companion itself), tuned to refract fairly well to a "point". The source of the light is the sun, and we will not get hotter than that. We do not need perfect point focus, but will gladly accept a central line of foci for various degrees of refraction, which generate -- you guessed it -- different colors of spotlight at different orbital distance of the large body from your planet's surface. Blue when it's close, red when it's far -- if it behaves like a proper lens-shaped lens. This would also result in color change as the effect sweeps from the edge of the home planet (farther) to the center (a bit closer). Warning signs would be similar to normal eclipses (the effect would only be observable from the very height of the eclipse). Finally, a combination of rotational and orbital planes for the three bodies involved can do wonders for making a simple periodic set of processes appear miserably non-periodic, particularly for observers located at different points on the surface of the home body.



            I'll create a graphic. But I think this thing is doable with a lot less machinery than has been proposed so far, and without violating physics to the point of ridicule.






            share|improve this answer



















            • 1




              The only reason this won't get ridiculed for violating physics is that most people don't know about conservation of etendue.
              – Mark
              yesterday










            • Mark, how so? A garden variety magnifying glass does not get as hot as the point focused upon.
              – Haakon Dahl
              7 mins ago














            1












            1








            1






            I haven't seen this directly addressed, so I'll pose it as an answer:



            TL;DR: Large moon with atmosphere refracts to a "point" on your planet. See below for etendue/thermodynamics, refraction, periodicity, and "warning signs".



            A companion (moon, twin planet, or even planet as primary with your story set on a comfortable moon of a gas giant) large enough to hold a substantial atmosphere can perhaps be tuned to get the result you need.



            Devise an atmosphere for the companion body with a powerful thermal inversion somewhere that reduces some of the spreading due to typical refraction of a density-stratified lens.



            So we effectively have a ring-shaped lens, fairly narrow (edge view of the companion's atmosphere) but of very wide diameter (the companion itself), tuned to refract fairly well to a "point". The source of the light is the sun, and we will not get hotter than that. We do not need perfect point focus, but will gladly accept a central line of foci for various degrees of refraction, which generate -- you guessed it -- different colors of spotlight at different orbital distance of the large body from your planet's surface. Blue when it's close, red when it's far -- if it behaves like a proper lens-shaped lens. This would also result in color change as the effect sweeps from the edge of the home planet (farther) to the center (a bit closer). Warning signs would be similar to normal eclipses (the effect would only be observable from the very height of the eclipse). Finally, a combination of rotational and orbital planes for the three bodies involved can do wonders for making a simple periodic set of processes appear miserably non-periodic, particularly for observers located at different points on the surface of the home body.



            I'll create a graphic. But I think this thing is doable with a lot less machinery than has been proposed so far, and without violating physics to the point of ridicule.






            share|improve this answer














            I haven't seen this directly addressed, so I'll pose it as an answer:



            TL;DR: Large moon with atmosphere refracts to a "point" on your planet. See below for etendue/thermodynamics, refraction, periodicity, and "warning signs".



            A companion (moon, twin planet, or even planet as primary with your story set on a comfortable moon of a gas giant) large enough to hold a substantial atmosphere can perhaps be tuned to get the result you need.



            Devise an atmosphere for the companion body with a powerful thermal inversion somewhere that reduces some of the spreading due to typical refraction of a density-stratified lens.



            So we effectively have a ring-shaped lens, fairly narrow (edge view of the companion's atmosphere) but of very wide diameter (the companion itself), tuned to refract fairly well to a "point". The source of the light is the sun, and we will not get hotter than that. We do not need perfect point focus, but will gladly accept a central line of foci for various degrees of refraction, which generate -- you guessed it -- different colors of spotlight at different orbital distance of the large body from your planet's surface. Blue when it's close, red when it's far -- if it behaves like a proper lens-shaped lens. This would also result in color change as the effect sweeps from the edge of the home planet (farther) to the center (a bit closer). Warning signs would be similar to normal eclipses (the effect would only be observable from the very height of the eclipse). Finally, a combination of rotational and orbital planes for the three bodies involved can do wonders for making a simple periodic set of processes appear miserably non-periodic, particularly for observers located at different points on the surface of the home body.



            I'll create a graphic. But I think this thing is doable with a lot less machinery than has been proposed so far, and without violating physics to the point of ridicule.







            share|improve this answer














            share|improve this answer



            share|improve this answer








            edited yesterday

























            answered yesterday









            Haakon DahlHaakon Dahl

            31315




            31315








            • 1




              The only reason this won't get ridiculed for violating physics is that most people don't know about conservation of etendue.
              – Mark
              yesterday










            • Mark, how so? A garden variety magnifying glass does not get as hot as the point focused upon.
              – Haakon Dahl
              7 mins ago














            • 1




              The only reason this won't get ridiculed for violating physics is that most people don't know about conservation of etendue.
              – Mark
              yesterday










            • Mark, how so? A garden variety magnifying glass does not get as hot as the point focused upon.
              – Haakon Dahl
              7 mins ago








            1




            1




            The only reason this won't get ridiculed for violating physics is that most people don't know about conservation of etendue.
            – Mark
            yesterday




            The only reason this won't get ridiculed for violating physics is that most people don't know about conservation of etendue.
            – Mark
            yesterday












            Mark, how so? A garden variety magnifying glass does not get as hot as the point focused upon.
            – Haakon Dahl
            7 mins ago




            Mark, how so? A garden variety magnifying glass does not get as hot as the point focused upon.
            – Haakon Dahl
            7 mins ago











            0














            The moon discussion referenced by @Phil Frost suggests part of the answer. A moon is too small so the lens-like body or phenomenon has to be big enough to cover all or most of the sky from the point of view of the target planet (which may itself be just a moon in a bigger system).



            The problem is coming up with a celestial lens. If you can solve that, the rest is just a question of placing the target planet and the radiation source at a suitable scale and proximity.



            A lens spotlight redirects light from a large area outside the "spot" so the first warning of the death ray's proximity would be a significant darkening, similar to a solar eclipse. In the distance you might see reflections from dust or clouds within the cone of concentrated light, so you can see if it's coming closer.






            share|improve this answer








            New contributor




            maxwellsdemon is a new contributor to this site. Take care in asking for clarification, commenting, and answering.
            Check out our Code of Conduct.


















            • You would also see the rim of the magnified sun start to appear from one side as the danger area approached, and could tell if it was going to pass you by or go right over you. But focusing light in such a way requires arrangements of matter too specifically contrived for nature to be a believable explanation. Your best bet would seem to be along the lines of neglectful precursors.
              – Christopher James Huff
              yesterday
















            0














            The moon discussion referenced by @Phil Frost suggests part of the answer. A moon is too small so the lens-like body or phenomenon has to be big enough to cover all or most of the sky from the point of view of the target planet (which may itself be just a moon in a bigger system).



            The problem is coming up with a celestial lens. If you can solve that, the rest is just a question of placing the target planet and the radiation source at a suitable scale and proximity.



            A lens spotlight redirects light from a large area outside the "spot" so the first warning of the death ray's proximity would be a significant darkening, similar to a solar eclipse. In the distance you might see reflections from dust or clouds within the cone of concentrated light, so you can see if it's coming closer.






            share|improve this answer








            New contributor




            maxwellsdemon is a new contributor to this site. Take care in asking for clarification, commenting, and answering.
            Check out our Code of Conduct.


















            • You would also see the rim of the magnified sun start to appear from one side as the danger area approached, and could tell if it was going to pass you by or go right over you. But focusing light in such a way requires arrangements of matter too specifically contrived for nature to be a believable explanation. Your best bet would seem to be along the lines of neglectful precursors.
              – Christopher James Huff
              yesterday














            0












            0








            0






            The moon discussion referenced by @Phil Frost suggests part of the answer. A moon is too small so the lens-like body or phenomenon has to be big enough to cover all or most of the sky from the point of view of the target planet (which may itself be just a moon in a bigger system).



            The problem is coming up with a celestial lens. If you can solve that, the rest is just a question of placing the target planet and the radiation source at a suitable scale and proximity.



            A lens spotlight redirects light from a large area outside the "spot" so the first warning of the death ray's proximity would be a significant darkening, similar to a solar eclipse. In the distance you might see reflections from dust or clouds within the cone of concentrated light, so you can see if it's coming closer.






            share|improve this answer








            New contributor




            maxwellsdemon is a new contributor to this site. Take care in asking for clarification, commenting, and answering.
            Check out our Code of Conduct.









            The moon discussion referenced by @Phil Frost suggests part of the answer. A moon is too small so the lens-like body or phenomenon has to be big enough to cover all or most of the sky from the point of view of the target planet (which may itself be just a moon in a bigger system).



            The problem is coming up with a celestial lens. If you can solve that, the rest is just a question of placing the target planet and the radiation source at a suitable scale and proximity.



            A lens spotlight redirects light from a large area outside the "spot" so the first warning of the death ray's proximity would be a significant darkening, similar to a solar eclipse. In the distance you might see reflections from dust or clouds within the cone of concentrated light, so you can see if it's coming closer.







            share|improve this answer








            New contributor




            maxwellsdemon is a new contributor to this site. Take care in asking for clarification, commenting, and answering.
            Check out our Code of Conduct.









            share|improve this answer



            share|improve this answer






            New contributor




            maxwellsdemon is a new contributor to this site. Take care in asking for clarification, commenting, and answering.
            Check out our Code of Conduct.









            answered yesterday









            maxwellsdemonmaxwellsdemon

            1




            1




            New contributor




            maxwellsdemon is a new contributor to this site. Take care in asking for clarification, commenting, and answering.
            Check out our Code of Conduct.





            New contributor





            maxwellsdemon is a new contributor to this site. Take care in asking for clarification, commenting, and answering.
            Check out our Code of Conduct.






            maxwellsdemon is a new contributor to this site. Take care in asking for clarification, commenting, and answering.
            Check out our Code of Conduct.












            • You would also see the rim of the magnified sun start to appear from one side as the danger area approached, and could tell if it was going to pass you by or go right over you. But focusing light in such a way requires arrangements of matter too specifically contrived for nature to be a believable explanation. Your best bet would seem to be along the lines of neglectful precursors.
              – Christopher James Huff
              yesterday


















            • You would also see the rim of the magnified sun start to appear from one side as the danger area approached, and could tell if it was going to pass you by or go right over you. But focusing light in such a way requires arrangements of matter too specifically contrived for nature to be a believable explanation. Your best bet would seem to be along the lines of neglectful precursors.
              – Christopher James Huff
              yesterday
















            You would also see the rim of the magnified sun start to appear from one side as the danger area approached, and could tell if it was going to pass you by or go right over you. But focusing light in such a way requires arrangements of matter too specifically contrived for nature to be a believable explanation. Your best bet would seem to be along the lines of neglectful precursors.
            – Christopher James Huff
            yesterday




            You would also see the rim of the magnified sun start to appear from one side as the danger area approached, and could tell if it was going to pass you by or go right over you. But focusing light in such a way requires arrangements of matter too specifically contrived for nature to be a believable explanation. Your best bet would seem to be along the lines of neglectful precursors.
            – Christopher James Huff
            yesterday











            0














            The system could be a binary system with a neutron star or black hole orbiting close to the main star but in an eccentric orbit that lasts a few days. When it draws close to the primary it pulls off huge masses of coronal gasses and causes massive incredibly intense solar flares. If these incidents happen at the same time as the planet is in the wrong part of the sky then you can expect some serious pyrotechnics to hit the day side of the planet for a few hours.



            The black hole/neutron star would most likely have been captured rather than be an original part of the system, explaining the eccentric orbit and any unusual spin needed etc.



            It doesn't take a big stretch to somehow say that the gravity and magnetic fields of the neutron star focuses the ejections into beams somehow. So every X hours you get massive beams of solar energy being fired in random directions. You can then get the variance by saying whether those beams hit your planet or not.






            share|improve this answer





















            • I think this is too deadly. The situation you describe is similar to a classic nova, and the 10,000-fold increase in solar output from one of those will quite handily sterilize a planet.
              – Mark
              12 hours ago
















            0














            The system could be a binary system with a neutron star or black hole orbiting close to the main star but in an eccentric orbit that lasts a few days. When it draws close to the primary it pulls off huge masses of coronal gasses and causes massive incredibly intense solar flares. If these incidents happen at the same time as the planet is in the wrong part of the sky then you can expect some serious pyrotechnics to hit the day side of the planet for a few hours.



            The black hole/neutron star would most likely have been captured rather than be an original part of the system, explaining the eccentric orbit and any unusual spin needed etc.



            It doesn't take a big stretch to somehow say that the gravity and magnetic fields of the neutron star focuses the ejections into beams somehow. So every X hours you get massive beams of solar energy being fired in random directions. You can then get the variance by saying whether those beams hit your planet or not.






            share|improve this answer





















            • I think this is too deadly. The situation you describe is similar to a classic nova, and the 10,000-fold increase in solar output from one of those will quite handily sterilize a planet.
              – Mark
              12 hours ago














            0












            0








            0






            The system could be a binary system with a neutron star or black hole orbiting close to the main star but in an eccentric orbit that lasts a few days. When it draws close to the primary it pulls off huge masses of coronal gasses and causes massive incredibly intense solar flares. If these incidents happen at the same time as the planet is in the wrong part of the sky then you can expect some serious pyrotechnics to hit the day side of the planet for a few hours.



            The black hole/neutron star would most likely have been captured rather than be an original part of the system, explaining the eccentric orbit and any unusual spin needed etc.



            It doesn't take a big stretch to somehow say that the gravity and magnetic fields of the neutron star focuses the ejections into beams somehow. So every X hours you get massive beams of solar energy being fired in random directions. You can then get the variance by saying whether those beams hit your planet or not.






            share|improve this answer












            The system could be a binary system with a neutron star or black hole orbiting close to the main star but in an eccentric orbit that lasts a few days. When it draws close to the primary it pulls off huge masses of coronal gasses and causes massive incredibly intense solar flares. If these incidents happen at the same time as the planet is in the wrong part of the sky then you can expect some serious pyrotechnics to hit the day side of the planet for a few hours.



            The black hole/neutron star would most likely have been captured rather than be an original part of the system, explaining the eccentric orbit and any unusual spin needed etc.



            It doesn't take a big stretch to somehow say that the gravity and magnetic fields of the neutron star focuses the ejections into beams somehow. So every X hours you get massive beams of solar energy being fired in random directions. You can then get the variance by saying whether those beams hit your planet or not.







            share|improve this answer












            share|improve this answer



            share|improve this answer










            answered 22 hours ago









            Tim BTim B

            59.7k23167286




            59.7k23167286












            • I think this is too deadly. The situation you describe is similar to a classic nova, and the 10,000-fold increase in solar output from one of those will quite handily sterilize a planet.
              – Mark
              12 hours ago


















            • I think this is too deadly. The situation you describe is similar to a classic nova, and the 10,000-fold increase in solar output from one of those will quite handily sterilize a planet.
              – Mark
              12 hours ago
















            I think this is too deadly. The situation you describe is similar to a classic nova, and the 10,000-fold increase in solar output from one of those will quite handily sterilize a planet.
            – Mark
            12 hours ago




            I think this is too deadly. The situation you describe is similar to a classic nova, and the 10,000-fold increase in solar output from one of those will quite handily sterilize a planet.
            – Mark
            12 hours ago











            -2














            Reverse Solar Eclipse.

            An orbiting Ocean, say a liquid helium moon of the correct size. Meaning, blocking the entire star, or larger. It could create a focusing and penumbra effect based on your need for destruction. Reinforce your physics.






            share|improve this answer










            New contributor




            SiGGER is a new contributor to this site. Take care in asking for clarification, commenting, and answering.
            Check out our Code of Conduct.














            • 2




              The correct size is roughly "fills the entire sky". See conservation of etendue, XKCD What-If #145, and the answers to this question.
              – Mark
              yesterday
















            -2














            Reverse Solar Eclipse.

            An orbiting Ocean, say a liquid helium moon of the correct size. Meaning, blocking the entire star, or larger. It could create a focusing and penumbra effect based on your need for destruction. Reinforce your physics.






            share|improve this answer










            New contributor




            SiGGER is a new contributor to this site. Take care in asking for clarification, commenting, and answering.
            Check out our Code of Conduct.














            • 2




              The correct size is roughly "fills the entire sky". See conservation of etendue, XKCD What-If #145, and the answers to this question.
              – Mark
              yesterday














            -2












            -2








            -2






            Reverse Solar Eclipse.

            An orbiting Ocean, say a liquid helium moon of the correct size. Meaning, blocking the entire star, or larger. It could create a focusing and penumbra effect based on your need for destruction. Reinforce your physics.






            share|improve this answer










            New contributor




            SiGGER is a new contributor to this site. Take care in asking for clarification, commenting, and answering.
            Check out our Code of Conduct.









            Reverse Solar Eclipse.

            An orbiting Ocean, say a liquid helium moon of the correct size. Meaning, blocking the entire star, or larger. It could create a focusing and penumbra effect based on your need for destruction. Reinforce your physics.







            share|improve this answer










            New contributor




            SiGGER is a new contributor to this site. Take care in asking for clarification, commenting, and answering.
            Check out our Code of Conduct.









            share|improve this answer



            share|improve this answer








            edited 21 hours ago





















            New contributor




            SiGGER is a new contributor to this site. Take care in asking for clarification, commenting, and answering.
            Check out our Code of Conduct.









            answered yesterday









            SiGGERSiGGER

            132




            132




            New contributor




            SiGGER is a new contributor to this site. Take care in asking for clarification, commenting, and answering.
            Check out our Code of Conduct.





            New contributor





            SiGGER is a new contributor to this site. Take care in asking for clarification, commenting, and answering.
            Check out our Code of Conduct.






            SiGGER is a new contributor to this site. Take care in asking for clarification, commenting, and answering.
            Check out our Code of Conduct.








            • 2




              The correct size is roughly "fills the entire sky". See conservation of etendue, XKCD What-If #145, and the answers to this question.
              – Mark
              yesterday














            • 2




              The correct size is roughly "fills the entire sky". See conservation of etendue, XKCD What-If #145, and the answers to this question.
              – Mark
              yesterday








            2




            2




            The correct size is roughly "fills the entire sky". See conservation of etendue, XKCD What-If #145, and the answers to this question.
            – Mark
            yesterday




            The correct size is roughly "fills the entire sky". See conservation of etendue, XKCD What-If #145, and the answers to this question.
            – Mark
            yesterday











            -3














            A transparent sphere works as a burning glass so a moon of (impossible) clear material should do the trick by concentrating the rays from the sun if it orbited at the right distance.



            Trouble is that absorbtion would eat most of the light if the diameter was more than a kilometer. A thin ice-shell might work, but good luck with explaining the origin (and stability!!) of that ;-)






            share|improve this answer








            New contributor




            Mads Horn is a new contributor to this site. Take care in asking for clarification, commenting, and answering.
            Check out our Code of Conduct.


















            • Welcome to Worldbuilding, Mads Horn! If you have a moment, please take the tour and visit the help center to learn more about the site. You may also find Worldbuilding Meta and The Sandbox useful. Here is a meta post on the culture and style of Worldbuilding.SE, just to help you understand our scope and methods, and how we do things here. Have fun!
              – Gryphon
              yesterday






            • 1




              A thin layer of ice may let enough light through, but it won't have any significant lensing effect, not to mention it wouldn't survive without collapsing spectacularly for more than a few days after being conjured at best, let alone form naturally in the first place.
              – John Dvorak
              yesterday






            • 1




              "If it orbited at the right distance": the focal distance of a ball lens is $f = nD / 4(n - 1)$, with n being the index of refraction of the material and D the diameter of the sphere. For glass, this works out at about 0.8 D, so that orbit must be very close to the surface. Not to mention that the focus lies on the optical axis, so that it wont fall on the surface unless the moon is in conjuction with the Sun. And ball lenses are horrible lenses, they won't focus the light in a nice focal spot.
              – AlexP
              yesterday








            • 6




              Won't work, for reasons more than just absorption. See Would a Moon made of water pose a threat to Earth during eclipses?
              – Phil Frost
              yesterday


















            -3














            A transparent sphere works as a burning glass so a moon of (impossible) clear material should do the trick by concentrating the rays from the sun if it orbited at the right distance.



            Trouble is that absorbtion would eat most of the light if the diameter was more than a kilometer. A thin ice-shell might work, but good luck with explaining the origin (and stability!!) of that ;-)






            share|improve this answer








            New contributor




            Mads Horn is a new contributor to this site. Take care in asking for clarification, commenting, and answering.
            Check out our Code of Conduct.


















            • Welcome to Worldbuilding, Mads Horn! If you have a moment, please take the tour and visit the help center to learn more about the site. You may also find Worldbuilding Meta and The Sandbox useful. Here is a meta post on the culture and style of Worldbuilding.SE, just to help you understand our scope and methods, and how we do things here. Have fun!
              – Gryphon
              yesterday






            • 1




              A thin layer of ice may let enough light through, but it won't have any significant lensing effect, not to mention it wouldn't survive without collapsing spectacularly for more than a few days after being conjured at best, let alone form naturally in the first place.
              – John Dvorak
              yesterday






            • 1




              "If it orbited at the right distance": the focal distance of a ball lens is $f = nD / 4(n - 1)$, with n being the index of refraction of the material and D the diameter of the sphere. For glass, this works out at about 0.8 D, so that orbit must be very close to the surface. Not to mention that the focus lies on the optical axis, so that it wont fall on the surface unless the moon is in conjuction with the Sun. And ball lenses are horrible lenses, they won't focus the light in a nice focal spot.
              – AlexP
              yesterday








            • 6




              Won't work, for reasons more than just absorption. See Would a Moon made of water pose a threat to Earth during eclipses?
              – Phil Frost
              yesterday
















            -3












            -3








            -3






            A transparent sphere works as a burning glass so a moon of (impossible) clear material should do the trick by concentrating the rays from the sun if it orbited at the right distance.



            Trouble is that absorbtion would eat most of the light if the diameter was more than a kilometer. A thin ice-shell might work, but good luck with explaining the origin (and stability!!) of that ;-)






            share|improve this answer








            New contributor




            Mads Horn is a new contributor to this site. Take care in asking for clarification, commenting, and answering.
            Check out our Code of Conduct.









            A transparent sphere works as a burning glass so a moon of (impossible) clear material should do the trick by concentrating the rays from the sun if it orbited at the right distance.



            Trouble is that absorbtion would eat most of the light if the diameter was more than a kilometer. A thin ice-shell might work, but good luck with explaining the origin (and stability!!) of that ;-)







            share|improve this answer








            New contributor




            Mads Horn is a new contributor to this site. Take care in asking for clarification, commenting, and answering.
            Check out our Code of Conduct.









            share|improve this answer



            share|improve this answer






            New contributor




            Mads Horn is a new contributor to this site. Take care in asking for clarification, commenting, and answering.
            Check out our Code of Conduct.









            answered yesterday









            Mads HornMads Horn

            31




            31




            New contributor




            Mads Horn is a new contributor to this site. Take care in asking for clarification, commenting, and answering.
            Check out our Code of Conduct.





            New contributor





            Mads Horn is a new contributor to this site. Take care in asking for clarification, commenting, and answering.
            Check out our Code of Conduct.






            Mads Horn is a new contributor to this site. Take care in asking for clarification, commenting, and answering.
            Check out our Code of Conduct.












            • Welcome to Worldbuilding, Mads Horn! If you have a moment, please take the tour and visit the help center to learn more about the site. You may also find Worldbuilding Meta and The Sandbox useful. Here is a meta post on the culture and style of Worldbuilding.SE, just to help you understand our scope and methods, and how we do things here. Have fun!
              – Gryphon
              yesterday






            • 1




              A thin layer of ice may let enough light through, but it won't have any significant lensing effect, not to mention it wouldn't survive without collapsing spectacularly for more than a few days after being conjured at best, let alone form naturally in the first place.
              – John Dvorak
              yesterday






            • 1




              "If it orbited at the right distance": the focal distance of a ball lens is $f = nD / 4(n - 1)$, with n being the index of refraction of the material and D the diameter of the sphere. For glass, this works out at about 0.8 D, so that orbit must be very close to the surface. Not to mention that the focus lies on the optical axis, so that it wont fall on the surface unless the moon is in conjuction with the Sun. And ball lenses are horrible lenses, they won't focus the light in a nice focal spot.
              – AlexP
              yesterday








            • 6




              Won't work, for reasons more than just absorption. See Would a Moon made of water pose a threat to Earth during eclipses?
              – Phil Frost
              yesterday




















            • Welcome to Worldbuilding, Mads Horn! If you have a moment, please take the tour and visit the help center to learn more about the site. You may also find Worldbuilding Meta and The Sandbox useful. Here is a meta post on the culture and style of Worldbuilding.SE, just to help you understand our scope and methods, and how we do things here. Have fun!
              – Gryphon
              yesterday






            • 1




              A thin layer of ice may let enough light through, but it won't have any significant lensing effect, not to mention it wouldn't survive without collapsing spectacularly for more than a few days after being conjured at best, let alone form naturally in the first place.
              – John Dvorak
              yesterday






            • 1




              "If it orbited at the right distance": the focal distance of a ball lens is $f = nD / 4(n - 1)$, with n being the index of refraction of the material and D the diameter of the sphere. For glass, this works out at about 0.8 D, so that orbit must be very close to the surface. Not to mention that the focus lies on the optical axis, so that it wont fall on the surface unless the moon is in conjuction with the Sun. And ball lenses are horrible lenses, they won't focus the light in a nice focal spot.
              – AlexP
              yesterday








            • 6




              Won't work, for reasons more than just absorption. See Would a Moon made of water pose a threat to Earth during eclipses?
              – Phil Frost
              yesterday


















            Welcome to Worldbuilding, Mads Horn! If you have a moment, please take the tour and visit the help center to learn more about the site. You may also find Worldbuilding Meta and The Sandbox useful. Here is a meta post on the culture and style of Worldbuilding.SE, just to help you understand our scope and methods, and how we do things here. Have fun!
            – Gryphon
            yesterday




            Welcome to Worldbuilding, Mads Horn! If you have a moment, please take the tour and visit the help center to learn more about the site. You may also find Worldbuilding Meta and The Sandbox useful. Here is a meta post on the culture and style of Worldbuilding.SE, just to help you understand our scope and methods, and how we do things here. Have fun!
            – Gryphon
            yesterday




            1




            1




            A thin layer of ice may let enough light through, but it won't have any significant lensing effect, not to mention it wouldn't survive without collapsing spectacularly for more than a few days after being conjured at best, let alone form naturally in the first place.
            – John Dvorak
            yesterday




            A thin layer of ice may let enough light through, but it won't have any significant lensing effect, not to mention it wouldn't survive without collapsing spectacularly for more than a few days after being conjured at best, let alone form naturally in the first place.
            – John Dvorak
            yesterday




            1




            1




            "If it orbited at the right distance": the focal distance of a ball lens is $f = nD / 4(n - 1)$, with n being the index of refraction of the material and D the diameter of the sphere. For glass, this works out at about 0.8 D, so that orbit must be very close to the surface. Not to mention that the focus lies on the optical axis, so that it wont fall on the surface unless the moon is in conjuction with the Sun. And ball lenses are horrible lenses, they won't focus the light in a nice focal spot.
            – AlexP
            yesterday






            "If it orbited at the right distance": the focal distance of a ball lens is $f = nD / 4(n - 1)$, with n being the index of refraction of the material and D the diameter of the sphere. For glass, this works out at about 0.8 D, so that orbit must be very close to the surface. Not to mention that the focus lies on the optical axis, so that it wont fall on the surface unless the moon is in conjuction with the Sun. And ball lenses are horrible lenses, they won't focus the light in a nice focal spot.
            – AlexP
            yesterday






            6




            6




            Won't work, for reasons more than just absorption. See Would a Moon made of water pose a threat to Earth during eclipses?
            – Phil Frost
            yesterday






            Won't work, for reasons more than just absorption. See Would a Moon made of water pose a threat to Earth during eclipses?
            – Phil Frost
            yesterday




















            draft saved

            draft discarded




















































            Thanks for contributing an answer to Worldbuilding Stack Exchange!


            • Please be sure to answer the question. Provide details and share your research!

            But avoid



            • Asking for help, clarification, or responding to other answers.

            • Making statements based on opinion; back them up with references or personal experience.


            Use MathJax to format equations. MathJax reference.


            To learn more, see our tips on writing great answers.





            Some of your past answers have not been well-received, and you're in danger of being blocked from answering.


            Please pay close attention to the following guidance:


            • Please be sure to answer the question. Provide details and share your research!

            But avoid



            • Asking for help, clarification, or responding to other answers.

            • Making statements based on opinion; back them up with references or personal experience.


            To learn more, see our tips on writing great answers.




            draft saved


            draft discarded














            StackExchange.ready(
            function () {
            StackExchange.openid.initPostLogin('.new-post-login', 'https%3a%2f%2fworldbuilding.stackexchange.com%2fquestions%2f135840%2fhow-can-a-planet-have-a-deadly-eclipse-like-spotlight%23new-answer', 'question_page');
            }
            );

            Post as a guest















            Required, but never shown





















































            Required, but never shown














            Required, but never shown












            Required, but never shown







            Required, but never shown

































            Required, but never shown














            Required, but never shown












            Required, but never shown







            Required, but never shown







            Popular posts from this blog

            An IMO inspired problem

            Management

            Investment